Burtonian line is seen with poisoning of

Gamma camera in Nuclear Medicine is used for:
A. Organ imaging
B. Measuring the radioactivity
C. Monitoring the surface contamination
D. RIA

Correct answer : B. Measuring the radioactivity

All of the following are the disadvantages of anesthetic ether, except:
A. Induction is slow
B. Irritant nature of ether increases salivary and bronchial secretions
C. Cautery cannot be used
D. Affects blood pressure and is liable to produce arrhythmias

Correct answer : D. Affects blood pressure and is liable to produce arrhythmias

Which one of the following is the fastest acting inhalational agent?
A. Halothane
B. Isoflurane
C. Ether
D. Sevoflurane

Correct answer : D. Sevoflurane

Sevoflurane is the fastest acting among the options given. Desflurane is THE fastest acting inhalational agent.

An 8 year old boy from Bihar presents with a 6 months history of an ill defined, hypopigmented slightly atrophic macule on the face. The most likely diagnosis is:
A. Pityriasis alba
B. Indeterminate leprosy
C. Morphea
D. Calcium deficiency

Correct answer : B. Indeterminate leprosy

Bihar is a known endemic region for leprosy.

All of the following drugs are effective in the treatment of pityriasis versicolor except:
A. Selenium sulphide
B. Ketoconazole
C. Griseofulvin
D. Clotrimazole

Correct answer : C. Griseofulvin

Griseofulvin is used in the treatment of dermatophyte infections.

A 16 year-old boy presented with asymptomatic, multiple, erythematous, annular lesions with a collarette of scales at the periphery of the lesions present on the trunk. The most likely diagnosis is:
A. Pityriasis versicolor
B. Pityriasis alba
C. Pityriasis rosea
D. Pityriasis rubra pilaris

Correct answer : C. Pityriasis rosea

The classical lesions of pityriasis rosea are described in the question.

The most common site of leak in CSF rhinorrhoea is:
A. Sphenoid sinus
B. Frontal sinus
C Cirbriform plate
D. Tegmen tympani

Correct answer : C Cirbriform plate

The most common and earliest manifestation of carcinoma of the glottis is:
A. Hoarseness
B. Haemoptysis
C. Cervical lymph nodes
D. Stridor

Correct answer : A. Hoarseness

Carcinoma glottis affects the mobility of vocal cord and produces hoarseness of voice. This helps in early identification of disease.

Use of seigel’s speculum during examination of the ear provides all except:
A. Magnification
B. Assessment of movement of the tympanic membrane
C. Removal of foreign body from the ear
D. As applicator for the powdered antibiotic of ear

Correct answer : C. Removal of foreign body from the ear

Uses of Seigel’s speculum

Assess movement of tympanic membrane
Introduce medicine into the middle ear
To have a magnified view of ear pathology
To assess vestibular function by fistula test

A 25 year old male gives history of sudden painless loss of vision in one eye for the past 2 weeks. There is no history of trauma. On examination the anterior segment is normal but there is no fundal glow. Which one of the following is the most likely cause?
A. Vitreous haemorrhage
B. Optic atrophy
C. Developmental cataract
D. Acute attack of angle closure glaucoma

Correct answer : A. Vitreous haemorrhage

Among the options given, only vitreous haemorrhage causes sudden painless loss of vision.

A 30 year old man has 6/5 vision each eye, unaided. His cycloplegic retinoscopy is +1.0 D sph. at 1 metre distance. His complaints are blurring of newsprint at 30 cm, that clears up in about two minutes. The most probable diagnosis is:
A. Hypermetropia
B. Presbyopia
C. Accommodative inertia
D. Cycloplegia

Correct answer : C. Accommodative inertia

His distant vision is normal. He has blurring of newsprint at 30cm distance for 2 minutes. This is due to accommodative inertia. His accommodation system takes some time to focus on near objects. (Normally it should take only 1 second).

Horner’s syndrome is characterized by all of the following except:
A. Miosis
B. Enopthalmos
C. Ptosis
D. Cycloplegia

Correct answer : D. Cycloplegia

Components of horner’s syndrome –

Ptosis
Miosis
Anhidrosis
Enophthalmos
Loss of ciliospinal reflex

Which one of the following is the investigation of choice for evaluation of suspected Perthes disease?
A. Plain X-ray
B. Ultrasonography (US)
C. Computed Tomography (CT)
D. Magnetic Resonance Imaging (MRI)

Correct answer : D. Magnetic Resonance Imaging (MRI)

MRI is the investigation of choice for Perthe’s disease

A 45 year old was given steroids after renal transplant. After 2 years he had difficulty in walking and pain in both hips. Which one of the following is most likely cause?
A. Primary Osteoarthritis
B. Avascular necrosis
C. Tuberculosis
D. Aluminum toxicity

Correct answer : B. Avascular necrosis

Mnemonic for causes of avascular necrosis other than fracture – SCLERA

S – Steroids
C – Caisson’s disease
LE – Lupus Erythematosus
R – Radiation
A – Alcohol

A 30 year old man had a road traffic accident and sustained fracture of femur. Two days later he developed sudden breathlessness. The most probable cause can be:
A. Pneumonia
B. Congestive heart failure
C. Bronchial asthma
D. Fat Embolism

Correct answer : D. Fat Embolism

The fat globules released from the bone marrow (due to trauma / orthopedic procedures) are deposited in the pulmonary capillary bed. This causes ischemia and inflammation. The symptoms appear 12-72 hours after the trauma. Initial symptoms are tachycardia and tachypnoea. There may be high spiking fever and non palpable petichae, especially over upper body. Neurological symptoms like agitation, delirium, stupor and coma can occur. (The fat globules may cross the pulmonary circulation via arteriovenous connections).

In an adult male, on per rectal examination, the following structures can be felt anteriorly except:
A. Internal iliac lymph nodes
B. Bulb of the penis
C. Prostate
D. Seminal vesicle when enlarged

Correct answer : A. Internal iliac lymph nodes

Internal iliac lymph nodes can be palpated laterally when they are enlarged, not anteriorly.

The following group of lymph nodes receives lymphatics from the uterus except:
A. External iliac
B. Internal iliac
C. Superficial inguinal
D. Deep inguinal

Correct answer : D. Deep inguinal

The most common type of total anomalous pulmonary venous connection is:
A. Supracardiac
B. Infracardiac
C. Mixed
D. Cardiac

Correct answer : A. Supracardiac

Eisenmenger syndrome is characterized by all except:
A. Return of left ventricle & right ventricle to normal size
B. Pulmonary veins not distended
C. Pruning of peripheral pulmonary arteries
D. Dilatation of central pulmonary arteries

Correct answer : A. Return of left ventricle & right ventricle to normal size

Right ventricular hypertrophy that develops in Eisenmenger syndrome will not return to normal size.

Diagnosis of beta Thalassemia is established by:
A. NESTROFT Test
B. HbA1c estimation
C. Hb electrophoresis
D. Target cells in peripheral smear

Correct answer : C. Hb electrophoresis

NESTROFT Test – Naked Eye Single Tube Red Cell Osmotic Fragility Test – used for screening for Thalassemia
HbA1c – Used to assess long term glycemic control in diabetics (blood sugar control over past 3 months)
Target cells – A feature of thalassemia, but not diagnostic

Infants of diabetic mother are likely to have the following cardiac anomaly:
A. Coarctation of aorta
B. Fallot’ s tetrology
C. Ebstein’s anomaly
D. Transposition of great arteries

Correct answer : D. Transposition of great arteries

Most common CVS anomalies in a child born to a diabetic mother : Transposition of great arteries, Ventricular septal defect

All are the risk factors associated with macrosomia except:
A. Maternal obesity
B. Prolonged Pregnancy
C. Previous large infant
D. Short Stature

Correct answer : D. Short Stature

All are the causes of intrauterine growth retardation except:
A. Anemia
B. Pregnancy induced hypertension
C. Maternal heart disease
D. Gestational diabetes

Correct answer : D. Gestational diabetes

Gestational diabetes is associated with fetal macrosomia. Overt diabetes can cause IUGR.

In which one of the following conditions is sialography contraindicated?
A, Ductal calculus
B. Chronic parotitis
C. Acute parotitis
D. Recurrent sialadenitis

Correct answer : C. Acute parotitis

Acute parotitis is a contraindication for saliography.

Abbey Estlander flap is used in the reconstruction of:
A. Buccal mucosa
B. Lip
C. Tongue
D. Palate

Correct answer : B. Lip

Abbey Estlander flap (stair step advancement technique) is used in reconstruction of lip.

The most common complication seen in hiatus hernia is:
A. Oesophagitis
B. Aspiration pneumonitis
C, Volvulus
D. Esophageal stricture

Correct answer : A. Oesophagitis

A 50 year old man, an alcoholic and a smoker presents with a 3 hour history of increasing shortness of breath. He started having this pain while eating, which was constant and radiated to the back and interscapular region. He was a known hypertensive. On examination, he was cold and clammy with a heart rate of 130/min, and a BP of 80/40 mm Hg. JVP was normal. All peripheral pulses were present and equal. Breath sounds were decreased at the left lung base and chest x-ray showed left pleural effusion. Which one of the following is the most likely diagnosis?
A. Acute aortic dissection
B. Acute myocardial infarction
C. Rupture of the esophagus
D. Acute pulmonary embolism

Correct answer : A. Acute aortic dissection

Chest pain radiating to interscapular area and signs of shock indicates the diagnosis of acute aortic dissection especially in those with risk factors like hypertension.

A 59 year old man with severe myxomatous mitral regurgitation is asymptomatic, with a left ventricular ejection fraction of 45% and an end-systolic diameter index of 2.9 cm/m2. The most appropriate treatment is:
A. Mitral valve repair or replacement
B. No treatment
C. ACE inhibitor therapy
D. Digoxin and diuretic therapy

Correct answer : A. Mitral valve repair or replacement

Surgical treatment of severe mitral regurgitation is recommended even if the person is asymptomatic when the LV ejection fraction drops below 60% and there is progressive LV dysfunction.

All of the following may cause ST segment elevation on EKG, except:
A. Early repolarization variant
B. Constrictive pericarditis
C. Ventricular aneurysm
D. Prinzmetal angina

Correct answer : B. Constrictive pericarditis

ST elevation can occur in acute pericarditis.

For the treatment of case a of class III dog bite, all of the following are correct except:
A. Give Immunoglobulins for passive immunity
B. Give ARV
C. Immediately stitch wound under antibiotic coverage
D. Immediately wash wound with soap and water

Correct answer : C. Immediately stitch wound under antibiotic coverage

Immediate stitching of bite wounds is contraindicated. It increases the trauma and increases the spread of virus into deeper tissues.

A 2-year old female child was brought to a PHC with a history of cough and fever for 4 days with inability to drink for last 12 hours. On examination, the child was having weight of 5 kg, and respiratory rate of 45/minute with fever. The child will be classified as suffering from:
A. Very severe disease
B. Severe Pneumonia
C. Pneumonia
D. No Pneumonia

Correct answer : A. Very severe disease

The child is unable to drink water for past 12 hours. It is a danger sign which indicates very severe disease.

All of the following are true about the Herd Immunity for infectious diseases except:
A. It refers to group protection beyond what is afforded by the protection of immunized individuals
B. lt is likely to be more for infections that do not have a sub clinical phase
C. It is affected by the presence and distribution of alternative animal hosts
D. In the case of tetanus it does not protect the individual

Correct answer : B. lt is likely to be more for infections that do not have a sub clinical phase

Herd immunity is more in case of infections with a sub clinical phase

Which of the following statements is not correct regarding diatoms?
A. Diatoms are aquatic unicellular plant
B. Diatoms have an extracellular coat composed of magnesium
C. Acid digestion technique is used to extract diatoms
D. Presence of diatoms in the femoral bone marrow is an indication of antemortem inhalation of water

Correct answer : B. Diatoms have an extracellular coat composed of magnesium

Diatoms have an extracellular coat of silica.

The most reliable criteria in Gustafson’s method of identification is:
A. Cementum apposition
B. Transparency of root
C. Attrition
D. Root resorption

Correct answer : B. Transparency of root

The most common pattern of fingerprint is:
A. Arch
B. Loop
C. Whorl
D. Composite

Correct answer : B. Loop

Fingerprint patterns and their relative frequency:

Loop – 67%
Whorls – 25%
Arches – 6-7%
Composite – 1-2%

Which one of the following drugs has been shown to offer protection from gastric aspiration syndrome in a patient with symptoms of reflux?
A. Ondansetron
B. Metoclopramide
C. Sodium citrate
D. Atropine

Correct answer : B. Metoclopramide

Metoclopramide protects against gastric aspiration by:

Increases tone of lower esophageal sphincter
Increasing rate of gastric emptying

Which one of the following drugs increases gastro-intestinal motility?
A. Glycopyrrolate
B, Atropine
C. Neostigmine
D. Fentanyl

Correct answer : C. Neostigmine

Acetyl choline stimulates gastrointestinal motility. Neostigmine is an acetyl cholinesterase inhibitor which increases the levels of acetylcholine.

With which of the following theophylline has an antagonistic interaction?
A. Histamine receptors
B. Bradykinin receptors
C. Adenosine receptors
D. lmidazoline receptors

Correct answer : C. Adenosine receptors

Theophylline is an antagonist at adenosine receptors.

A child was diagnosed to be suffering from diarrhoea due to Compylobacter jejuni. Which of the following will be the correct environmental conditions of incubation of the culture plates of the stool sample:
A. Temperature of 42°C and microaerophilic
B. Temperature of 42°C and 10% carbon dioxide
C. Temperature of 37°C and microaerophilic
D. Temperature of 37°C and 10% carbon dioxide

Correct answer : A. Temperature of 42°C and microaerophilic

Culture conditions for Compylobacter jejuni

Low oxygen tension (5%)
Increased carbon dioxide levels (10%)
Incubation temperature of 42°C

A bacterial disease that has been associated with the 3 “Rs” i.e., rats, ricefields, and rainfall is:
A. Leptospirosis
B. Plague
C. Melioidosis
D. Rodent bite fever

Correct answer : A. Leptospirosis

Rats are important reservoirs. Farmers working in the rice fields are at increased risk of infection. Disease transmission occurs when rain water contaminated with urine of rats gets into contact with abraded skin.

With reference to infections with Escherichia coli the following are true except:
A. Enteroaggregative E. coli is associated with Persistant diarrhoea
B. Enterohemorrhagic E. coli can cause haemolytic uraemic syndrome
C. Enteroinvasive E. coli produces a disease similar to salmonellosis
D. Enterotoxigenic E. coli is a common cause of travelers’ diarrhoea

Correct answer : C. Enteroinvasive E. coli produces a disease similar to salmonellosis

Enteroinvasive E. coli produces a disease similar to shigellosis. (Invasive nature)

All endothelial cells produce thrombomodulin except those found in:
A. Hepatic circulation
B. Cutaneous circulation
C. Cerebral microcirculation
D. Renal circulation

Correct answer : C. Cerebral microcirculation

In-situ DNA nick end labeling can quantitate:
A. Fraction of cells in apoptotic pathways
B. Fraction of cells in S phase
C. p53 gene product
D. bcr/abl gene

Correct answer : A. Fraction of cells in apoptotic pathways

In apoptosis, Apaf-1 is activated by release of which of the following substances from the mitochondria?
A. Bcl 2
B. Bax
C. Bcl-XL
D. Cytochrome C

Correct answer : D. Cytochrome C

All of the following factors normally increase the length of the ventricular cardiac muscle fibers except:
A. Increased venous tone
B. Increased total blood volume
C. Increased negative intrathoracic pressure
D. Lying to standing change in posture

Correct answer : D. Lying to standing change in posture

When the person stands, there is pooling of blood in the legs. This causes decrease in venous return and decrease in end diastolic volume ( and length of ventricular cardiac muscle fibers).

Distribution of blood flow in mainly regulated by the;
A. Arteries
B. Arterioles
C. Capillaries
D. Venules

Correct answer : B. Arterioles

Arterioles are the major site of peripheral vascular resistance.

The vasodilatation produced by carbon dioxide is maximum in one of the following:
A. Kidney
B. Brain
C. Liver
D. Heart

Correct answer : B. Brain

Vasodilatation effect of CO2 is maximum in skin and brain.

All of the following statements regarding vas deferens are true except:
A. The terminal part is dilated to form ampulla
B. It crosses ureter in the region of ischial spine
C. lt passes lateral to inferior epigastric artery at deep inguinal ring
D. It is separated from the base of bladder by the peritoneum

Correct answer : D. It is separated from the base of bladder by the peritoneum

There is no peritoneal layer between vas deferens and the bladder.

The Prostatic urethra is characterized by all of the following features, except that it:
A. ls the widest and most dilatable part
B. Presents a concavity posteriorly
C. Lies closer to anterior surface of prostate
D. Receives Prostatic ductules along its posterior wall

Correct answer : B. Presents a concavity posteriorly

Prostatic urethra is concave anteriorly.

Benign Prostatic hypertrophy results in obstruction of the urinary tract. The specific condition is associated with enlargement of the:
A. Entire prostate gland
B. Lateral lobes
C. Median lobe
D. Posterior lobes

Correct answer : C. Median lobe

Median lobe of prostate is prone to development of BPH.
Posterior lobe of prostate is prone to development of malignancy.

The most common leukocytoclastic vasculitis affecting children is:
A. Takayasu disease
B. Mucocutaneous lymph node syndrome (Kawasaki disease)
C. Henoch–Schönlein purpura
D. Polyarteritis nodosa

Correct answer : C. Henoch–Schönlein purpura

HSP is the most common childhood vasculitis. It is produces leukocytoclastic vasculitis.

The coagulation profile in a 13-year old girl with Menorrhagia having von Willebrands disease is:
A. Isolated prolonged PTT with a normal PT
B. Isolated prolonged PT with a normal PTT
C. Prolongation of both PT and PTT
D. Prolongation of thrombin time

Correct answer : A. Isolated prolonged PTT with a normal PT

In VWF, there is impairment of intrinsic coagulation pathway. Hence there is isolated prolonged PTT.

Bart‘s hydrops fetalis is lethal because:
A. Hb Bart’s cannot bind oxygen
B. The excess alpha globin form insoluble precipitates
C. Hb Bart’s cannot release oxygen to fetal tssues
D. Microcytic red cells become trapped in the placenta

Correct answer : C. Hb Bart’s cannot release oxygen to fetal tssues

Hb Barts is formed of 4 gamma chains. The oxygen affinity is so high that it releases very little oxygen into the fetal tissues.

The smallest diameter of the true pelvis is:
A. Interspinous diameter
B. Diagonal conjugate
C. True conjugate
D. Intertuberous diameter

Correct answer : A. Interspinous diameter

Interspinous diameter – 10.5cm is the smallest among the options provided.

Which one of the following biochemical parameters is the most sensitive to detect open spina bifida?
A. Maternal serum alpha fetoprotein
B. Amniotic fluid alpha fetoprotein
C. Amniotic fluid acetyl cholinesterase
D. Amniotic fluid glucohexaminase

Correct answer : C. Amniotic fluid acetyl cholinesterase

The best period of gestation to carry out chorion villous biopsy for prenatal diagnosis is?
A. 8-10 weeks
B. 10-12 weeks
C. 12-14 weeks
D. 14-16 weeks

Correct answer : B. 10-12 weeks

In which one of the following head and neck cancer, perineural invasion is most commonly seen?
A. Adenocarcinoma
B. Adenoid cystic carcinoma
C. Basal Cell Adenoma
D. Squamous cell carcinoma

Correct answer : B. Adenoid cystic carcinoma

Perineural invasion is characteristically seen in adenoid cystic carcinoma. It is the most common malignant tumour of submandibular salivary glands.

Lumbar sympathectomy is of value in the management of:
A. Intermittent claudication
B. Distal ischaemia affecting the skin of the toes
C. Arteriovenous Fistula
D. Back pain

Correct answer : B. Distal ischaemia affecting the skin of the toes

The sympathetic system controls the blood flow to skin (not muscle). Sympathetic tone has a vasoconstrictor effect. Hence lumbar sympathectomy will cause vasodilation and increased blood flow to skin of toes.

A blood stained discharge from the nipple indicates one of the following:
A. Breast abscess
B. Fibroadenoma
C. Duct Papilloma
D. Fat Necrosis of Breast

Correct answer : C. Duct Papilloma

Causes of blood stained nipple discharge

Intraductal papilloma
Carcinoma
Duct ectasia
Fibrocystic disease

Palpable purpura could occur in the following conditions, except:
A. Thrombocytopenia
B. Small-vessel vasculitis
C. Gonococcal infection
D. Acute meningococcemia

Correct answer : A. Thrombocytopenia

Palpable purpura does not occur in thrombocytopenia.

Causes of palpable purpura

Rocky mountain spotted fever
Acute meningococcemia
Disseminated gonococcal infection
Ecthyma gangrenosum
Henoch Schonlein purpura
Polyarteritis nodosa
Leucocytoclastic vasculitis

All of the following are the causes of relative polycythemia except:
A. Dehydration
B. Dengue haemorrhagic fever
C. Gaisbock syndrome
D. High altitude

Correct answer : D. High altitude

People living at high altitudes have absolute polycythemia due to increased erythropoietin synthesis.

In which of the following conditions Splenectomy is not useful?
A. Herediatary spherocytosis
B. Porphyria
C. Thalassemia
D. Sickle cell disease with large spleen

Correct answer : B. Porphyria

Splenectomy has little role in the management of porphyria

‘Endemic Disease` means that a disease:
A. Occurs clearly in excess of normal expectancy
B. ls constantly present in a given population group
C. Exhibits seasonal pattem
D. Is prevalent among animals

Correct answer : B. ls constantly present in a given population group

An endemic disease is constantly present in a given population group.
Epidemic disease occurs clearly in excess of normal expectancy

Which one of the following is a good index of the severity of an acute disease?
A. Cause specific death rate
B. Case fatality rate
C. Standardized mortality ratio
D. Five year survivial

Correct answer : B. Case fatality rate

Case fatality rate = (Deaths due to the particular disease / Total number of cases of the particular disease) X 100
It represents the killing power of the disease.

Infant mortality does not include:
A. Early neonatal mortality
B. Perinatal mortality
C. Post neonatal mortality
D. Late neonatal mortality

Correct answer : B. Perinatal mortality

Infant mortality includes death within the first year of life. Perinatal mortality includes late foetal deaths (28 weeks of gestation to birth) and early neonatal deaths (birth to 7 days).

Other terms:
Late neonatal mortality – 7 days to 28 days
Neonatal mortality – birth to 28 days (early neonatal mortality + late neonatal mortality)
Post neonatal mortality – 28 days to 1 year

The minimum age at which an individual is responsible for his criminal act is:
A. 7 years
B. 12 years
C. 16 years
D. 21 years

Correct answer : A. 7 years

An individual below the age of 7 years will not be responsible for his criminal act. (Sec. 82 IPC)

The most reliable method of identification of an individual is:
A. Dactylography
B. Scars
C. Anthropometry
D. Handwriting

Correct answer : A. Dactylography

Fingerprinting is a very useful method in idenfication of a person. A fingerprint is unique to an individual and is permanent. The best method is DNA fingerprinting.

In India, magistrate inquest is done in the following cases except:
A. Exhumation cases
B. Dowry deaths within 5 years of marriage
C. Murder cases
D. Death of a person in police custody

Correct answer : C. Murder cases

Cases in which magistrate’s inquest is held:

Dowry death
Exhumation
Death in police custody
Death in a psychiatric hospital
Death in jail
Death as a result of police shooting

Which one of the following is true of adrenal suppression due to steroid therapy?
A. It is not associated with atrophy of the adrenal glands
B. It does not occur in patients receiving inhaled steroids
C. It should be expected in anyone receiving > 5mg Prednisolone daily
D. Following cessation, the stress response normalizes after 8 weeks

Correct answer : C. It should be expected in anyone receiving > 5mg Prednisolone daily

Steroid therapy can cause adrenal suppression with atrophy of adrenal gland. It can occur even with inhaled steroids. Following cessation of steroid intake, the stress response may take 1 year or more to normalize.

All of the following hormones have cell surface receptors except:
A. Adrenaline
B. Growth Hormone
C. Insulin
D. Thyroxine

Correct answer : D. Thyroxine

Steroid hormones have cytoplasmic receptors and thyroid hormones have intranuclear receptors.

A highly ionized drug:
A. Is excreted mainly by the kidney
B. Can cross the placental barrier easily
C. ls well absorbed from the intestine
D. Accumulates in the cellular lipids

Correct answer : A. Is excreted mainly by the kidney

Highly ionized drugs are lipid insoluble. Hence their movement across cell membrane is decreased. (Resulting in decreased absorption, decreased movement across placenta, decreased reabsorption in kidney etc…)

The following statements are true regarding Clostridium perfringens except:
A. lt is the commonest cause of gas gangrene
B. It is normally present in human faeces
C. The principal toxin of C. perfringens is the alpha toxin
D. Gas gangrene producing strains of C. perfringens produce heat resistant spores

Correct answer : D. Gas gangrene producing strains of C. perfringens produce heat resistant spores

C. perfringens spores are not resistant to heat. They can be destroyed by boiling.

All of the following Vibrio sp. are halophilic, except:
A. V. cholerae
B. V. parahaemolyticus
C. V. alginolyticus
D. V. fluvialis

Correct answer : A. V. cholerae

V. cholerae is non halophilic.

In the small intestine, cholera toxin acts by:
A. ADP-ribosylation of the G regulatory protein
B. Inhibition of adenyl cyclase
C. Activation of GTPase
D. Active absorption of NaCl

Correct answer : A. ADP-ribosylation of the G regulatory protein

Cholera toxin increases adenyl cyclase activity by irreversible ADP ribosylation of GTP binding domain of adenyl cyclase.

Fibrinoid necrosis may be observed in all of the following, except:
A. Malignant hypertension
B. Polyarteritis nodosa
C. Diabetic glomerulosclerosis
D. Aschoff’s nodule

Correct answer : C. Diabetic glomerulosclerosis

Fibrinoid necrosis is not described in diabetic glomerulosclerosis.

All of the following vascular changes are observed in acute inflammation, except:
A. Vasodilation
B. Stasis of blood
C. Increased vascular permeability
D. Decreased hydrostatic pressure

Correct answer : D. Decreased hydrostatic pressure

Acute inflammation is characterised by increase in hydrostatic pressure. (Due to vasodilation, there is increased blood flow which increases the hydrostatic pressure)

All of the following statements are true regarding reversible cell injury, except:
A. Formation of amorphous densities in the mitochondrial matrix
B. Diminished generation of adenosine triphosphate (ATP)
C. Formation of blebs in the plasma membrane
D. Detachment of ribosomes from the granular endoplasmic reticulum

Correct answer : A. Formation of amorphous densities in the mitochondrial matrix

It is a feature of irreversible cell injury.

Fluoride, used in the collection of blood samples for glucose estimation, inhibits the enzyme:
A. Glucokinase
B. Hexokinase
C. Enolase
D. Glucose-6-phosphatase

Correct answer : C. Enolase

Flouride is used to prevent glycolysis in the blood sample.

Enzymes that move a molecular group from one molecule to another are known as:
A. Ligases
B. Oxido—reductases
C. Transferases
D. Dipeptidases

Correct answer : C. Transferases

Enzyme that transfer functional groups from one molecule to another are called transferases.

The amino acid residue having an imino side chain is:
A. Lysine
B. Histidine
C. Tyrosine
D. Proline

Correct answer : D. Proline

Proline has an imino group (NH) instead of amino (NH2).

CO2 is primarily transported in the arterial blood as:
A. Dissolved CO2
B. Carbonic Acid
C. Carbaminohemoglobin
D. Bicarbonate

Correct answer : D. Bicarbonate

CO2 transport in blood

Bicarbonate – 70%
Carbaminohemoglobin – 23%
Dissolved CO2 – 7%

In which of the following a reduction in arterial oxygen tension occurs?
A. Anaemia
B. CO poisoning
C. Moderate exercise
D. Hypoventilation

Correct answer : D. Hypoventilation

Reduction in arterial oxygen tension occurs in respiratory failure – Type 1 (decreased oxygenation) or Type 2 (defective ventillation).

Pulmonary surfactant is secreted by:
A. Type I pneumocytes
B. Type II pneumocytes
C. Clara cells
D. Bronchial epithelial cells

Correct answer : B. Type II pneumocytes

Surfactant is produced by Type II pneumocytes.

While doing thoracocentesis, it is advisable to introduce needle along:
A. Upper border of the rib
B. Lower border of the rib
C. In the center of the intercostal space
D. In the anterior part of the intercostal space

Correct answer : A. Upper border of the rib

The intercostal neurovascular bundle lies along the lower boder of the rib. Hence we should introduce the needle along the upper border of rib to prevent damage to the neurovascular bundle.

The femoral ring is bounded by the following structures except:
A. Femoral vein
B. Inguinal ligament
C. Femoral artery
D. Lacunar ligament

Correct answer : C. Femoral artery

Boundaries of femoral ring:

Anterior – Inguinal ligament
Medial – Lacunar ligament
Posterior – Pectineus and covering fascia
Lateral – Septum separating it from femoral vein

The carpal tunnel contains all of the following important structures except?
A. Median Nerve
B. Flexor pollicis longus
C. Flexor carpi radialis
D. Flexor digitorum superficialis

Correct answer : C. Flexor carpi radialis

Contents of carpal tunnel:

Median nerve
Flexor pollicis longus tendon
Flexor digitorum profundus tendons – 4
Flexor digitorum superficialis tendons – 4

A total of 10 structures pass through the carpal tunnel.

Polar bodies are formed during:
A. Spermatogenesis
B. Organogenesis
C. Oogenesis
D. Morphogenesis

Correct answer : C. Oogenesis

All of the following are the components of the white pulp of spleen, except:
A. Periarteriolar lymphoid sheath
B. B cells
C. Antigen presenting cells
D. Vascular sinus

Correct answer : D. Vascular sinus

Vascular sinuses are seen in red pulp of spleen.

A 5 year old patient is scheduled for tonsillectomy. On the day of surgery he had running nose, temperature 37.5°C and dry cough. Which of the following should be the most appropriate decision for surgery?
A. Surgery should be cancelled
B. Can proceed for surgery if chest is clear and there is no history of asthma
C. Should get Xray chest before proceeding for surgery
D. Cancel surgery for 3 weeks and patient to be put on antibiotics

Correct answer : D. Cancel surgery for 3 weeks and patient to be put on antibiotics

Upper respiratory tract infection is a contraindication for tonsillectomy. So the surgery should be cancelled and the patient should be put on antibiotics.

Which one of the following statements is incorrect about Optic glioma?
A. Has a peak incidence in first decade
B. Arises from oligodendrocytes
C. Causes meningeal hyperplasia
D. Is associated with type I neurofibromatosis

Correct answer : C. Causes meningeal hyperplasia

Features of optic glioma:

Cells of origin – astrocytes, oligodendrocytes of optic nerve
Peak incidence – 2 to 5 years
Optic glioma is not known to cause meningeal hyperplasia.

An affected male infant born to normal parents could be an example of all of the following, except
A. An Autosomal dominant disorder
B. An Autosomal recessive disorder
C. A polygenic disorder
D. A vertically transmitted disorder

Correct answer : A. An Autosomal dominant disorder

An autosomal dominant disorder if present in the parents will always express itself phenotypically. So it is not possible for normal parents to have an child affected with an autosomal dominant disorder.

An affected male infant born to normal parents could be an example of all of the following, except
A. An Autosomal dominant disorder
B. An Autosomal recessive disorder
C. A polygenic disorder
D. A vertically transmitted disorder

Correct answer : A. An Autosomal dominant disorder

An autosomal dominant disorder if present in the parents will always express itself phenotypically. So it is not possible for normal parents to have an child affected with an autosomal dominant disorder.

A child with a small head, minor anomalies of the face including a thin upper lip, growth delay, and developmental disability can have all of the following, except:
A. A chromosomal syndrome
B. A teratogenic syndrome
C. A mendelian syndrome
D. A polygenic syndrome

Correct answer : D. A polygenic syndrome

The features given can occur as a part of a chromosomal / teratogenic / mendelian syndrome. Polygenic inheritance is the answer of exclusion. In a polygenic inheritance, multiple genes are involved in the phenotypic expression. Some examples of polygenic inheritance are hypertension and diabetes.

The process underlying differences in expression of a gene according to which parent has transmitted is called:
A. Anticipation
B. Mosaicism
C. Non penetrance
D. Genomic imprinting

Correct answer : D. Genomic imprinting

In genomic imprinting, a gene / group of genes on the paternal or maternal chromosome is selectively inactivated. The functional allele will be provided by the counterpart.
Example:

Angelman syndrome – deletion of maternal gene
Prader Willi syndrome – deletion of paternal gene

The following complications during pregnancy increase the risk of postpartum hemorrhage (PPH) except:
A. Hypertension
B. Macrosomia
C. Twin pregnancy
D. Hydramnios

Correct answer : A. Hypertension

Causes of postpartum haemorrhage:

Atonicity
Trauma
Coagulopathy
Retained placenta and placental fragments
Morbidly adherent placenta
Uterine inversion

A malignant tumor of childhood, that metastasizes to bones most often is:
A. Wilm’s tumor
B. Neuroblastoma
C. Adrenal gland tumors
D. Granulosa cell tumor of ovary

Correct answer : B. Neuroblastoma

60-70% of metastasis in case of neuroblastoma are skeletal metastasis.

A primigravida presents to casualty at 32 weeks gestation with acute pain abdomen for 2 hours, vaginal bleeding and decreased fetal movements. She should be managed by;
A. Immediate cesarean section
B. Immediate induction of labor
C. Tocolytic therapy
D. Magnesium sulphate therapy

Correct answer : A. Immediate cesarean section

Vaginal bleeding at 32 weeks with acute abdominal pain and decreased fetal movements suggests the diagnosis of abruption. Cesarean section should be done immediately to prevent fetal death and development of maternal complications.

The most preferred approach for pituitary surgery at the present time is:
A. Transcranial
B. Transethmoidal
C. Trans-sphenoidal
D. Transcallosal

Correct answer : C. Trans-sphenoidal

Which of the following is not a preferred site for planning vascular access for maintenance hemodialysis?
A. Nondominant extremity
B. Upper limb
C. Radiocephalic AV fistula
D. Saphenofemoral fistula

Correct answer : D. Saphenofemoral fistula

Usual site is forearm of non dominant hand. The femoral vessels are used only if all upper limb vessles have been used.

Dacron vascular graft is:
A. Nontextile synthetic
B. Textile synthetic
C. Nontextile biologic
D. Textile biologic

Correct answer : B. Textile synthetic

Dacron is a synthetic graft material. It is a type of medical textile.

Renal artery stenosis may occur in all of the following except:
A. Atherosclerosis
B. Fibromuscular dysplasia
C. Takayasu’s arteritis
D. Polyarteritis nodosa

Correct answer : D. Polyarteritis nodosa

Causes of renal artery stenosis:

Atherosclerosis
Giant cell arteritis
Fibromuscular dysplasia
Takayasu arteritis
Tranplant renal artery stenosis
Antiphospholipid antibody syndrome

The presence of anti Saccharomyces cerevisae antibody is a surrogate marker of one of the following:
A. Celiac disease
B. Crohn’s disease
C. Ulcerative colitis
D. Tropical sprue

Correct answer : B. Crohn’s disease

Saccharomyces cerevisae antibody positivity:

Chron’s disease – 60-70%
Ulcerative colitis – 10-15%

Anticentromere antibodies are most commonly associated with:
A. Diffuse cutaneous systemic sclerosis
B. Mixed connective tissue disease
C. CREST syndrome
D. Polymyositis

Correct answer : C. CREST syndrome

Anticentromere antibodies are seen mostly in those with CREST syndrome. (40-80% patients with CREST syndrome are anticentromere antibody positive)

All of the following are part of the treatment of lithium toxicity, except:
A. Treating dehydration
B. Ingestion of polystyrene sulfonate
C. Hemodialysis
D. Using an antagonist

Correct answer : D. Using an antagonist

There is no known antagonist for lithium.

Pretreatment evaluation for lithium therapy should include:
A. Fasting blood sugar
B. Serum creatinine
C. Liver function tests
D. Platelet count

Correct answer : B. Serum creatinine

Pretreatment evaluation for lithium therapy

Serum creatinine
Thyroid function test
ECG
Complete blood count
Serum electrolytes
Pregnancy test

Nevirapine is a
A. Protease inhibitor
B. Nucleoside reverse transcriptase inhibitor
C. Non nucleoside reverse transcriptase inhibitor
D. Fusion inhibitor

Correct answer : C. Non nucleoside reverse transcriptase inhibitor

Which of the following statements is incorrect about phthisis bulbi?
A. The intraocular pressure is increased
B. Calcification of the lens is common
C. Sclera is thickened
D. Size of the globe is reduced

Correct answer : A. The intraocular pressure is increased

Intraocular pressure is decreased in phthisis bulbi.

Which one of the following stromal dystrophy is a recessive condition?
A. Lattice dystrophy
B, Granular dystrophy
C. Macular dystrophy
D. Fleck dystrophy

Correct answer : C. Macular dystrophy

It is autosomal recessive

The most common retrobulbar orbital mass in adults is:
A. Neurofibroma
B. Meningioma
C. Cavernous hemangioma
D. Schwannoma

Correct answer : C. Cavernous hemangioma

The most important determinant of prognosis in Wilms tumor:
A. Stage of disease
B. Loss of heterozygosity of chromsome lp
C. Histology
D. Age less than one year at presentation

Correct answer : C. Histology

Histology and stage of disease are important prognostic factors. But histology is more important.

A 1 month old boy is referred for failure to thrive. On examination, he shows feature of congestive heart failure. The femoral pulses are feeble as compared to branchial pulses. The most likely clinical diagnosis is:
A. Congenital aortic stenosis
B. Coarctation of aorta
C. Patent ductus arteriosus
D. Congenital aortoiliac disease

Correct answer : B. Coarctation of aorta

Feeble femoral pulses compared to brachial pulse indicates coarctation of aorta.

ln which of the following conditions left atrium is not enlarged:
A. Ventricular septal defect
B. Atrial septal defect
C. Aortopulmonary window
D. Patent ductus arteriosus

Correct answer : B. Atrial septal defect

Right atrium and ventricle is enlarged in ASD. Left atrium can enlarge once Eisenmenger’s syndrome develops.

Allen’s test is useful in evaluating:
A. Thoracic outlet compression
B. Presence of cervical rib
C. Integrity of palmar arch
D. Digital blood flow

Correct answer : C. Integrity of palmar arch

How to perform allen’s test: (To check for patency of radial and ulnar arteries and to confirm integrity of the palmar arterial arch between them)

The patient makes a tight fist
The examiner compresses the radial and ulnar arteries distally
The fist is released and compression of the ulnar artery is released
The speed of return of colour to the palm is looked for
Steps 1 and 2 are repeated, but release the radial artery compression instead of the ulnar artery
Again look for speed of return of colour to the palm

Normally, the colour should return in 5-7seconds. This indicates that the arteries and the palmar arch is patent and functioning well.

The most common cause of acquired arteriovenous fistula is;
A. Bacterial infection
B. Fungal infection
C. Blunt trauma
D. Penetrating trauma

Correct answer : D. Penetrating trauma

Which of the following statements is true regarding testicular tumors?
A. Are embryonal cell carcinomas in 95% of cases
B. Bilateral in up to 10% cases
C. Teratomas are more common than seminomas
D. Usually present after 50 years of age

Correct answer : B. Bilateral in up to 10% cases

15% of familial and 5% of sporadic cases of testicular tumour are bilateral.

Drug induced lupus can be identified by:
A. Anti histone antibodies
B. Double stranded DNA antibodies
C. Antinuclear antibodies
D. Anti SM antibodies

Correct answer : A. Anti histone antibodies

Anti histone antibodies are present in more than 95% individuals with drug induced lupus.

Which of the following is the most common presenting symptom of non cirrhotic portal hypertension?
A. Chronic liver failure
B. Ascites
C. Upper gastrointestinal bleeding
D. Encephalopathy

Correct answer : C. Upper gastrointestinal bleeding

Acute gastrointestinal bleed is the most common presenting symptom of extrahepatic portal hypertension.

Which of the following is not a common feature of Anorexia Nervosa?
A. Binge eating
B. Amenorrhoea
C. Self perception of being ‘fat`
D. Under weight

Correct answer : A. Binge eating

Binge eating is a common feature of bulimia. It is rarely seen in anorexia nervosa.

All of the following drugs are protease inhibitors except:
A. Nelfinavir
B. Saquinavir
C. Abacavir
D. Ritonavir

Correct answer : C. Abacavir

Abacavir is a nucleoside reverse transcriptase inhibitor

One of the following statements regarding mycophenolate mofetil is incorrect:
A. lt is a prodrug
B. lt is a selective uncompetitive and reversible inhibitor
C. It also inhibits calcineurin
D. Selectively inhibits lymphocyte proliferation

Correct answer : C. It also inhibits calcineurin

Mycophenolate mofetil inhibits inosine monophosphate dehydrogenase. It has no action on calcineurin.

A diabetic patient developed cellulitis due to staphylococcus aureus, which was found to be Methicillin resistant on the antibiotic sensitivity testing. All the following antibiotics will be appropriate except:
A. Vancomycin
B. Imipenem
C. Teicoplanin
D. Linezolid

Correct answer : B. Imipenem

Most strains of MRSA are not sensitive to Imipenem

All of the following enzymes are regulated by calcium or calmodulin, except:
A. Adenylate cyclase
B. Glycogen synthase
C. Guanylyl cyclase
D. Hexokinase

Correct answer : D. Hexokinase

Some enzymes regulated by calcium / calmodulin:

Adenylate cyclase
Guanylate cyclase
Pyruvate kinase
Pyruvate dehydrogenase
Pyruvate carboxylase
Phospholipase A2
Glycogen synthase
Glycerol 3 phosphate dehydrogenase

Vitamin B12 acts as co-enzyme to which one of the following enzymes?
A. Isocitrate dehydrogenase
B. Homocysteine methyl transferase
C. Glycogen synthase
D. G-6-P dehydrogenase

Correct answer : B. Homocysteine methyl transferase

It catalyses the conversion of homocystine to methionine.

The following methods can be used to detect the point mutation in the beta (B)-globin gene that causes sickle cell anemia, except:
A. Polymerase chain reaction with allele-specific oligonucleotide hybridization
B. Southem blot analysis
C. DNA sequencing
D. Northem blot analysis

Correct answer : D. Northem blot analysis

Point mutation in a gene is identified by checking for defects in the DNA. Northern blot is used for analysis of RNA.

In which of the following uveitic conditions is it contraindicated to put in an intraocular lens after cataract surgery?
A. Fuch’s heterochromic cyclitis
B. Juvenile rheumatoid arthritis
C. Psoriatic arthritis
D. Reiter’s syndrome

Correct answer : B. Juvenile rheumatoid arthritis (JRA)

There is a high incidence of cataract in those with JRA. But implantation of intraocular lens following cataract surgery is associated with high risk for development of fibrosis, secondary retrolental membranes and lens capture.

Typically bilateral inferior subluxation of lens is seen in:
A. Marfan’s syndrome
B. Homocystinuria
C. Hyperinsulinemia
D. Ocular trauma

Correct answer : B. Homocystinuria

Bilateral subluxation of lens:

Marfan’s syndrome – upwards and outwards
Homocystinuria – downwards and inwards

The most common cause of vitreous hemorrhage in adults is:
A. Retinal hole
B. Trauma
C. Hypertension
D. Diabetes

Correct answer : D. Diabetes

Proliferative diabetic retinopathy is the most common cause of spontaneous vitreous haemorrhage in adults.

Blalock and Taussig shunt is done between:
A. Aorta to pulmonary artery
B. Aorta to pulmonary vein
C. Subclavian artery to pulmonary vein
D. Subclavian vein to artery

Correct answer : A. Aorta to pulmonary artery

Blalock and Taussig shunt is used in the surgical management of tetralogy of fallot. It is shunts blood from the subclavian artery to the pulmonary artery. That is not given among the list of options. As subclavian artery is a branch of aorta, the best answer would be ‘aorta to pulmonary artery.’

The following features are true for tetralogy of Fallot, except:
A. Ventricular septal defect
B. Right ventricular hypertrophy
C. Atrial septal defect
D. Pulmonary stenosis

Correct answer : C. Atrial septal defect

Components of Tetralogy of Fallot:

Pulmonary stenosis
Overriding aorta
Ventricular septal defect
Right ventricular hypertrophy

Late onset hemorrhagic disease of newborn is characterized by all of the following features except:
A. Usually occurs in cow-milk fed babies
B. Onset occurs at 4-12 week of age
C. lntracranial hemorrhage can occur
D. Intramuscular vitamin K prophylaxis at birth has a protective role

Correct answer : A. Usually occurs in cow-milk fed babies

Haemorrhagic disease of newborn is more common in breast milk fed babies. (Breast milk is a poor source of Vitamin K)

Which vitamin deficiency is most commonly seen in a pregnant mother who is on phenytoin therapy for epilepsy?
A. Vitamin B6
B. Vitamin B12
C. Vitamin A
D. Folic acid

Correct answer : D. Folic acid

Folic acid absorption is decreased and its excretion is increased in those using phenytoin. Other vitamin deficiencies seen in phenytoin users : Vitamin D, Vitamin K. Hence pregnant woman on phenytoin should take folic acid, cholecalciferol and phytonadione supplements.

Which of the following tumors is not commonly known to increase in pregnancy?
A. Glioma
B. Pituitary adenoma
C. Meningioma
D. Neurofibroma

Correct answer : A. Glioma

Tumours which increase in size during pregnancy:

Pituitary adenoma
Neurofibroma
Meningioma
Angioma

Which of the following ovarian tumor is most prone to undergo torsion during pregnancy?
A. Serous cystadenoma
B. Mucinous cystadenoma
C. Dermoid cyst
D. Theca lutein cyst

Correct answer : C. Dermoid cyst

Dermoid cyst the mot common tumour to undergo torsion during pregnancy.

The treatment of choice for squamous cell anal cancer is:
A. Abdominoperineal resection
B. Laser fulgaration
C. Chemoradiotherapy
D. Platinum—based chemotherapy

Correct answer : C. Chemoradiotherapy

Chemotherapy with radiotherapy (Nigro regimen) is the preferred treatment for anal carcinoma.

Which of the following drugs is not a part of the ‘Triple Therapy’ immunosuppression for post-renal transplant patients?
A. Cyclosporine
B. Azathioprine
C. FK 506
D. Prednisolone

Correct answer : C. FK 506

Components of triple therapy immunosuppression for post-renal transplant patients are :

Cyclosporine
Azathioprine
Prednisolone

The most common cause of gastric outlet obstruction in India is:
A. Tuberculosis
B. Cancer of stomach
C. Duodenal lymphoma
D. Peptic ulcer disease

Correct answer : B. Cancer of stomach

Smoking is not a risk factor for development of?
A. Small cell carcinoma
B. Respiratory bronchiolitis
C. Emphysema
D. Bronchiolitis obliterans organizing pneumonia

Correct answer : D. Bronchiolitis obliterans organizing pneumonia

Smoking is a known risk factor for small cell carcinoma, respiratory bronchiolitis and emphysema.

Mycotic abscesses are due to:
A. Bacterial infection
B. Fungal infection
C. Viral infection
D. Mixed infection

Correct answer : B. Fungal infection

Mycotic aneurysm is an aneurysm infected because of:
A. Fungal infection
B. Blood-borne infection (intravascular)
C. Infection introduced from outside (extravascular)
D. Both intravascular and extravascular infection

Correct answer : D. Both intravascular and extravascular infection

The source of infection in a mycotic aneurysm can be intravascular or extravascular.

Which one ofthe following drugs is an antipseudomonal penicillin?
A. Cephalexin
B. Cloxacillin
C. Piperacillin
D. Dicloxacillin

Correct answer : C. Piperacillin

Antipseudomonal penicillins

Carboxypenicillins
    Carbenicillin
    Ticracillin
Ureidopenicillins
    Piperacillin
    Mezlocillin

All of the following are therapeutic uses of penicillin G, except:
A. Bacterial meningitis
B. Rickettsial infection
C. Syphilis
D. Anthrax

Correct answer : B. Rickettsial infection

Rickettsiae are immune to penicillin G.

Which of the following medications is contraindicated in patients with allergy to sulphonamides?
A. Levobunolol
B. Bimatoprost
C. Brinzolamide
D. Brimonidine

Correct answer : C. Brinzolamide

Brinzolamide is a carbonic anhydrase inhibitor and a sulphonamide derivative. So it cannot be used in those with allergy to sulphonamides. Others belonging to the same group are acetazolamide and dorzolamide.

Base substitution mutations can have the following molecular consequence except:
A. Changes one codon for an amino acid into another codon for that same amino acid
B. Codon for one amino acid is changed into a codon of another amino acid
C. Reading frame changes downstream to the mutant site
D. Codon for one amino acid is changed into a translation termination codon

Correct answer : C. Reading frame changes downstream to the mutant site

Reading frame shift can occur only in case of frame shift mutation – insertion or deletion. Base substitution does not produce reading frame shift.

All of the following cell types contain the enzyme telomerase which protects the length of telomeres at the end of chromosomes, except:
A. Germinal
B. Somatic
C. Hemopoietic
D. Tumor

Correct answer : B. Somatic

The somatic cells progressively lose DNA in the absence of telomerase and hence eventually undergo apoptosis.

During replication of DNA, which one of the following enzymes polymerizes the Okazaki fragments?
A. DNA Polymerase I
B. DNA Polymerase II
C. DNA Polymerase III
D. RNA Polymerase I

Correct answer : C. DNA Polymerase III

For the synthesis of the lagging strand, first an RNA primer is created. Then DNA polymerase III binds to the RNA primer and adds deoxyribonucleotides as short segments – Okazaki fragments.

The MR imaging in multiple sclerosis will show lesions in:
A. White matter
B. Grey matter
C. Thalamus
D. Basal ganglia

Correct answer : A. White matter

Multiple sclerosis is a demyelinating disorder. Hence the lesions will be seen in the white matter (due to loss of myelin).

The most important sign of signficant renal artery stenosis in an angiogram is:
A. A percentage diameter stenosis > 70%
B. Presence of collaterals
C. A systolic pressure gradient > 20 mm Hg across the lesion
D. Poststenotic dilatation of the renal artery

Correct answer : B. Presence of collaterals

Presence of collaterals indicate that there is significant renal artery stenosis.

The most accurate investigation for assessing ventricular function is:
A. Multislice CT
B. Echocardiography
C. Nuclear scan
D. MRI

Correct answer : B. Echocardiography

Which of the following agents is not used to provide induced hypotension during surgery?
A. Sodium nitroprusside
B. Hydralazine
C. Mephentermine
D. Esmolol

Correct answer : C. Mephentermine

Mephentermine is acts as an agonist on alpha and beta adrenergic receptors. This causes increased in cardiac output and blood pressure. Hence it cannot be used to induce hypotension.

Which of the following agents is used for the treatment of postoperative shivering?
A. Thiopentone
B. Suxamethonium
C. Atropine
D. Pethidine

Correct answer : D. Pethidine

Management of postoperative shivering:

Drug of choice – Tramadol
Alternate drugs – pethidine, pentazocine
Supportive treatment – oxygen inhalation (there is increased oxygen consumption during shivering)

Which of the following intravenous induction agents is the most suitable for day care surgery?
A. Morphine
B. Ketamine
C. Propofol
D. Diazepam

Correct answer : C. Propofol

Propofol is the induction agent of choice in day care surgery

Early induction
Smooth recovery
Decreased incidence of post operative nausea and vomiting

When should a case of Non-Insulin dependent diabetes mellitus (NIDDM) with a history of diabetes for one year have an ophthalmic examination?
A. As early as feasible
B. After 5 years
C. After 10 years
D. Only after visual symptoms develop

Correct answer : A. As early as feasible

The person may have been diabetic for some time before the diagnosis of NIDDM. Retinopathy may have already occured. Hence, ophthalmological testing should be done as early as possible after diagnosis of NIDDM. If there are no pathologic changes, he should come for review every 6 months / 1 year.

But in case of IDDM (Insulin dependant diabetes mellitus), the patients are diagnosed early in life. There is much less chance of any ophthalmologic disease at the time of diagnosis. Hence ophthalmologic examination is recommended 5 years after the diagnosis of IDDM.

Which drug can cause macular toxicity when given intravitreally?
A. Gentamicin
B. Vancomycin
C. Dexamethasone
D. Ceftazidime

Correct answer : A. Gentamicin

Vancomycin and Ceftazidime can cause macular toxicity when given in high doses. But macular toxicity has been reported with even low dose of gentamycin.

Which of the following drugs is not used topically for treatment of open angle glaucoma:
A. Latanoprost
B. Brimonidine
C. Acetazolamide
D. Dorzolamide

Correct answer : C. Acetazolamide

Acetazolamide is a carbonic anhydrase inhibitor. It is given orally and parenterally. It is not used topically. Dorzolamide is a topically active carbonic anhydrase inhibitor.

With reference to mumps which of the following is true?
A. Meningoencephalitis can precede parotitis
B. Salivary gland involvement is limited to the parotids
C. The patient is not infectious prior to clinical parotid enlargement
D. Mumps orchitis frequency leads to infertility

Correct answer : A. Meningoencephalitis can precede parotitis

Meningoencephalitis can occur 1 week before onset of parotitis.

The earliest indicator of response after starting iron in a 6-year-old girl with iron deficiency is:
A. Increased reticulocyte count
B. Increased hemoglobin
C. Increased ferritin
D. Increased serum iron

Correct answer : A. Increased reticulocyte count

Sequence of changes during correction of iron deficiency:

Clinical improvement in the child (increase in appetite, improvement in irritability)
Inital bone marrow response
Increased reticulocyte count
Haemoglobin levels return to normal
Body iron stores return to normal (ferritin levels)

The most common etiological agent for acute bronchiolitis in infancy is:
A. Influenza virus
B. Para influenza virus
C. Rhinovirus
D. Respiratory syncytial virus

Correct answer : D. Respiratory syncytial virus (It is responsible for more than 50% cases of bronchiolitis)

All of the following mechanisms might account for a reduced risk of upper genital tract infection in users of progestin releasing IUDs, except:
A. Reduced retrograde menstruation
B. Decreased ovulation
C. Thickened cervical mucus
D. Decidual changes in the endometrium

Correct answer : B. Decreased ovulation

Factors decreasing risk of upper genital tract infection in users of progestin releasing intrauterine contraceptive device:

Reduced retrograde menstruation – prevents seeding of fallopian tube with bacteria
Thickened cervical mucus – decreases bacterial penetration
Decidual changes in the endometrium – making it less susceptible to infection

Prenatal diagnosis at 16 weeks of pregnancy can be performed using all of the following, except:
A. Amniotic fluid
B. Maternal blood
C. Chorionic villi
D. Fetal blood

Correct answer : D. Fetal blood

Fetal blood sampling is done by cordocentesis. It is done in the 18-24 weeks gestational age.

Emergency contraception prevents pregnancy by all of the following mechanisms, except:
A. Delaying/inhibiting ovulation
B. Inhibiting fertilization
C. Preventing implantation of the fertilized egg
D. Interrupting an early pregnancy

Correct answer : D. Interrupting an early pregnancy

Emergency contraception is used to prevent pregnancy. Interruption of an early pregnancy is not contraception – it is termination of pregnancy.

All of the following modalities can be used for in situ ablation of liver secondaries, except?
A. Ultrasonic waves
B. Cryotherapy
C. Alcohol
D. Radiofrequency

Correct answer : A. Ultrasonic waves

Ablation of liver secondaries can be done by:

Cryotherapy
Radiofrequency ablation
Microwave ablation
Injection of chemicals like alcohol and acetic acid

Biliary stricture developing after laparascopic cholecystectomy usually occurs at which part of the common bile duct?
A. Upper
B. Middle
C. Lower
D. All sides with equal frequency

Correct answer : A. Upper

During surgery, the common bile duct may be mistakenly identified as the cystic duct. Hence, surgeon may ligate the common bile duct instead of the cystic duct resulting in the development of a stricture in the upper part.

Which one of the following statements is incorrect regarding stones in the common bile duct?
A. Can present with Charcot‘s triad
B. Are suggested by a bile duct diameter > 6 mm in ultrasound
C. ERCP, sphincterotomy and balloon clearance is now the standard treatment
D. When removed by exploration of the common bile ducts the T-tube can be removed after 3 days

Correct answer : D. When removed by exploration of the common bile ducts the T-tube can be removed after 3 days

T tubes are kept in the bile duct for 7-30 days. It allows track to mature and helps prevent bile leak and biliary peritonitis.

Which of the following is least likely to cause infective endocarditis?
A. Staphylococcus albus
B. Streptococcus faecalis
C. Salmonella typhi
D. Pseudomonas aeruginosa

Correct answer : C. Salmonella typhi

Infective endocarditis is a rare complication of Salmonella typhi infection.

Sudden cardiac death may occur in all of the following except:
A. Dilated cardiomyopathy
B. Hypertrophic cardiomyopathy
C. Eisenmengers syndrome
D. Ventricular septal defect

Correct answer : D. Ventricular septal defect

VSD is the best answer among the options given

All of the following are true for mitral valve prolapse, except:
A. Transmission may be as an autosomal dominant trait
B. Majority of the cases present with features of mitral regurgitation
C. The valve leaflets characteristically show myxomatous degeneration
D. The disease is one of the common cardiovascular manifestations of Marfan’s Syndrome

Correct answer : B. Majority of the cases present with features of mitral regurgitation

Features of Mitral valve prolapse:

Majority of patients are asymptomatic
Some present with non specific chest pain, dyspnoea, fatigue or palpitation
Myxomatous degeneration of heart valves is seen (mostly mitral valve)
Autosomal dominant pattern of inheritance
Prominent cardiovascular manifestation of Marfans syndrome
Mitral regurgitation can occur as a complication due to rupture of chordae tendinae or failure of papillary muscles

All of the following antibacterial agents acts by inhibiting cell wall synthesis, except:
A. Carbapenems
B. Monobactams
C. Cephamycins
D. Nitrofurantoin

Correct answer : D. Nitrofurantoin

Nitrofurantoin produces highly reactive intermediates which cause DNA damage. It does not inhibit cell wall synthesis.

The most common side effect reported in case of treatment with haloperidol is:
A. Hypotension
B. Akathisia
C. Dryness of mouth
D. Tic disorder

Correct answer : B. Akathisia

Haloperidol produces extrapyramidal side effects (Akathasia, Acute dystonia, Parkinsonism).

All are side effects of Clozapine, except;
A. Granulocytopenia
B. Seizures
C. Sedation
D. Extrapyramidal side effects

Correct answer : D. Extrapyramidal side effects

Side effects of Clozapine:

Agranulocytosis
Seizures
Sedation
Urinary incontinence
Weight gain
BP changes
Tachycardia

Clozapine is an atypical antipsychotic – hence the extrapyramidal side effects are minimal

Which one of the following is the complementary sequence of 5′ TTAAGCGTAC 3′?
A. 5’ GTACGCTTAA 3′
B. 5’ AATTCGCATG 3‘
C. 5‘ CATGCGAATT 3’
D. 5’ TTAAGCGTAC 3’

Correct answer : A. 5’ GTACGCTTAA 3′

How to calculate the complementary sequence of DNA:

Adenine binds with Thymine
Guanine binds with Cytosine
In the sequence given, make the following changes
    Replace A with T
    Replace T with A
    Replace C with G
    Replace G with C
    Replace 5′ with 3′
    Replace 3′ with 5′
That gives you the complementary sequence of DNA!

Note: Using the above method, we get the complementary DNA sequence as 3′ AATTCGCATG 5′. This is the same as 5′ GTACGCTTAA 3′ – which corresponds to option A.

The sigma subunit of prokaryotic RNA polymerase:
A. Binds the antibiotic rifampicin
B. Is inhibited by tx-amanitin
C. Specifically recognizes the promoter site
D. ls part of the core enzyme

Correct answer : C. Specifically recognizes the promoter site

Sigma subunit is responsible for recognition of promoter sites in the DNA.

Which type of RNA has the highest percentage of modified base?
A. mRNA
B. tRNA
C. rRNA
D. snRNA

Correct answer : B. tRNA

In tRNA, the bases (Adenine, Guanine, Uracil and Cytosine) are modified by methylation, reduction, deamination and rearrangement of glycosidic bonds.

All of the following development events are dependent on the production of maternal or fetal glucocorticoid, except:
A. Induction of thymic involution
B. Production of surfactant by type II alveolar cells
C. Functional thyroid
D. Functional hypothalamopituitary axis

Correct answer : C. Functional thyroid

Functions of maternal / fetal corticosteroids in the fetus:

Synthesis of surfactant in the lungs
Induction of thymic involution
Proper development of hypothalamopituitary axis

Ovulation is primarily caused by preovulatory surge of :
A. Estradiol
B. Luteinizing hormone
C. Progesterone
D. Follicle stimulating hormone

Correct answer : B. Luteinizing hormone

Ovulation occurs within 24-48 hours after the beginning of the LH surge.

Testosterone production is mainly contributed by:
A. Leydig cells
B. Sertoli cells
C. Seminiferous tubules
D. Epididymis

Correct answer : A. Leydig cells (factual question)

The procedure of choice for the evaluation of an aneurysm is:
A. Ultrasonography
B. Computed tomography
C. Magnetic resonance imaging
D. Arteriography

Correct answer : D. Arteriography

Which is the objective sign of identifying pulmonary plethora in a chest radiograph?
A. Diameter of the main pulmonary artery > 16 mm
B. Diameter of the left pulmonary artery > 16 mm
C. Diameter of the descending right pulmonary artery > 16 mm
D. Diameter of the descending left pulmonary artery > 16 mm

Correct answer : C. Diameter of the descending right pulmonary artery > 16 mm

Which of the following causes rib-notching in a chest radiograph?
A. Bidirectional Glenn shunt
B. Modified Blalock Taussig shunt
C. IVC occlusion
D. Coarctation of aorta

Correct answer : D. Coarctation of aorta

Causes of rib notching:

Superior rib notching
    Polio
    Restrictive lung disease
    Neurofibromatosis
    Connective tissue disease
    Osteogenesis imperfecta
    Hyperparathyroidism
Inferior rib notching
    Thrombosis of aorta
    Coarctation of arota
    Blalock Taussig shunt
    Occlusion of subclavian artery
    Pulmonary AV malformation

Which of the following anesthetic agents does not trigger malignant hyperthermia?
A. Halothane
B. Isoflurane
C. Suxamethonium
D. Thiopentone

Correct answer : D. Thiopentone

Anesthetic agents implicated in malignant hyperthermia:

Succinyl choline
Halothane
Isoflurane

Which of the following statements is not true about etomidate?
A. It is an intravenous anesthetic
B. It precipitates coronary insufficiency
C. It inhibits cortisol synthesis
D. It causes pain at site of injection

Correct answer : B. It precipitates coronary insufficiency

Disadvantages of etomidate:

Decrease cortisol synthesis
Painful injection
Nausea and vomiting during recovery period
Produce restlessness and rigidity

Which one of the following agents sensitizes the myocardium to catecholamines?
A. Isoflurane
B. Ether
C. Halothane
D. Propofol

Correct answer : C. Halothane

Halothane sensitizes the myocardium to the action of catecholamines and can produce severe ventricular arrhythmia.

Which of the following antiglaucoma medications can cause drowsiness?
A. Latanoprost
B. Timolol
C. Brimonidine
D. Dorzolamide

Correct answer : C. Brimonidine

Drowsiness, BP changes and Fatigue are the known side effects of Brimonidine.
Related Articles

anaesthesia mcq Anaesthesia – MCQ 2
anatomy mcq Anatomy – MCQ 64 – Injury to the long thoracic nerve
spm mcq SPM – MCQ 9 – Strategies for prevention of Neonatal Tetanus
pharmacology mcq Furosemide in acute pulmonary edema

Add a Comment

Your email address will not be published. Comments will be displayed only after moderation.

Comment:*

Name:*

Email Address:*

xpansile lytic osseous metastases are characteristic of primary malignancy of:
A. Kidney
B. Bronchus
C. Breast
D. Prostate

Correct answer : A. Kidney

Primary tumours producing osteolytic metastasis:

Kidney
Lung
Thyroid
GIT
Breast

Which of the following malignant tumors is radioresistant?
A. Ewing’s sarcoma
B. Retinoblastoma
C. Osteosarcoma
D. Neuroblastoma

Correct answer : C. Osteosarcoma

Osteosarcoma is highly radioresistant

The sodium content of ReSoMal (rehydration solution for malnourished children) is:
A. 90 mmol/L
B. 60 mmol/L
C. 45 mmol/L
D. 30 mmol/L

Correct answer : C. 45 mmol/L

Components of of ReSoMal:

Glucose 125 mmol/l
Sodium 45 mmol/l
Potassium 40 mmol/l
Chloride 70 mmol/l
Magnesium 3 mmol/l
Zinc 0.3 mmol/l
Copper 0.045 mmol/l
Citrate 7 mmol/l

The requirement of potassium is a child is:
A. 1-2 mEq/kg
B. 4-7 mEq/kg
C. 10-12 mEq/kg
D. 13-14 mEq/kg

Correct answer : A. 1-2 mEq/kg

Daily recommended intake of potassium is 1-2mEq/kg.

Which of the following malformation in a newborn is specific for maternal insulin dependent diabetes mellitus?
A. Transposition of great arteries
B. Caudal regression
C. Holoprosencephaly
D. Meningmyelocele

Correct answer : B. Caudal regression

Caudal regression is the most specific anomaly in a child born to a diabetic mother.

Which of the following statements is incorrect regarding levonorgestrel releasing intrauterine system:
A. There is increased incidence of menorrhagia
B. This system can be used as hormone replacement therapy
C. This method is useful for the treatment of endometerial hyperplasia
D. Irregular uterine bleeding can be problem initially

Correct answer : A. There is increased incidence of menorrhagia

The most common cause of tubal block in lndia is
A. Gonorrhea infection
B. Chlamydia infection
C. Tuberculosis
D. Bacterial vaginosis

Correct answer : C. Tuberculosis

All of the following mechanisms of action of oral contraceptive pill are true, except:
A. Inhibition of ovulation
B. Prevention of fertilization
C. Interference with implantation of fertilized ovum
D. Interference with placental function

Correct answer : D. Interference with placental function

Mechanism of action of oral contraceptive pills:

Prevents ovulation by inhibition of LH surge
Induce atrophic changes in endometrium, preventing nidation
Thickening of cervical mucus preventing entry of sperms
Increases tubal motility, so that the fertilized egg reaches uterine cavity before the endometrium is receptive for implantation

A patient of post cholecystectomy biliary stricture has undergone an ERCP three days ago. Following this she has developed acute cholangitis. The most likely organism is:
A. Escherichia coli
B. Bacillus fragilis
C. Streptococcus viridans
D. Pseudomonas aeruginosa

Correct answer : A. Escherichia coli

Apart from Escherichia coli. the other most common organism implicated in acute suppurative bacterial peritonitis is:
A. Bacteroides
B. Klebsiella
C. Peptostreptococcus
D. Pseudomonas

Correct answer : A. Bacteroides

Referred pain from all of the following conditions may be felt along the inner side of right thigh, except:
A. Inflamed pelvic appendix
B. Inflamed ovaries
C. Stone in pelvic ureter
D. Pelvic abscess

Correct answer : D. Pelvic abscess

All of the following may be seen in patients of cardiac tamponade except:
A. Kussmaul’s sign
B. Pulsus paradoxus
C. Electrical alternans
D. Right ventricular diastolic collapse in echocardiography

Correct answer : A. Kussmaul’s sign

Kussmaul’s sign is seen in constrictive pericarditis.

All of the following may occur due to hyperkalemia except:
A. Prolonged PR interval
B. Prolonged QRS interval
C. Prolonged QT interval
D. Ventricular asystole

Correct answer : C. Prolonged QT interval

All of the following are risk factors for atherosclerosis except:
A. Increased waist hip ratio
B. Hyperhomocysteinemia
C. Decreased fibrinogen levels
D. Decreased HDL levels

Correct answer : C. Decreased fibrinogen levels

Atherosclerosis risk is increased with increase in fibrinogen levels.

The common side effect with Fluoxetine therapy is:
A. Seizure
B. Anxiety
C. Hypotension
D. Loose stools

Correct answer : D. Loose stools

Which one of the following drugs does not interfere with folic acid metabolism?
A. Phenytoin
B. Gabapentin
C. Phenobarbitone
D. Primidone

Correct answer : B. Gabapentin

Concomitant administration of clonazepam with which of the following antiepileptic drug can precipitate absence status?
A. Sodium valproate
B. Phenobarbitone
C. Carbamazepine
D. Phenytoin

Correct answer : A. Sodium valproate

The percentage of pulmonary emboli, that proceed to infarction is approximately:
A. 0-5%
B. 5-15%
C. 20-30%
D. 30-40%

Correct answer : B. 5-15%

All of the following statements regarding subendocardial infarction are true except?
A. These are multifocal in nature
B. These often result from hypotension or shock
C. Epicarditis is not seen
D. These may result in aneurysm

Correct answer : D. These may result in aneurysm

Ventricular aneurysms occur as a result of transmural infarcts. In subendocardial infarction, less than half of the ventricular wall thickness undergoes infarction.

Neointimal hyperplasia causes vascular graft failure as a result of hypertrophy of:
A. Endothelial cells
B. Collagen fibers
C. Smooth muscle cells
D. Elastic fibers

Correct answer : C. Smooth muscle cells

Mitochondrial DNA is:
A. Closed circular
B. Nicked circular
C. Linear
D. Open circular

Correct answer : A. Closed circular

Microsatellite sequence is:
A. Small satellite
B. Extra chromosomal DNA
C. Short sequence (2-5) repeat DNA
D. Looped-DNA

Correct answer : C. Short sequence (2-5) repeat DNA

Euchromatin is the region of DNA that is relatively:
A. Uncondensed
B. Condensed
C. Overcondensed
D. Partially condensed

Correct answer : A. Uncondensed

Calcitonin is secreted by;
A. Thyroid gland
B. Parathyroid gland
C. Adrenal glands
D. Ovaries

Correct answer : A. Thyroid gland

Which of the following organs is not involved in calcium homeostasis?
A. Kidneys
B. Skin
C. Intestines
D. Lungs

Correct answer : D. Lungs

Kidneys – Filtration, reabsorption and excretion of calcium. Also involved in activation of Vitamin D
Skin – Vitamin D syntheisis
Intestines – Absorption of calcium from diet

The prime driving force for counter current multiplier system is:
A. Medullary hyperosmolarity
B. Reabsorption of Na+ in thick ascending limb
C. Action of ADH via aquaporin channels
D. Urea recycling

Correct answer : B. Reabsorption of Na+ in thick ascending limb

The primary driving force is reabsorption of Na+ in thick ascending limb. Medullary hyperosmolarity is an effect produced by the countercurrent multiplier system.

Rivastigmine and Donepezil are drugs used predominantly in the management of:
A. Depression
B. Dissociation
C. Delusions
D. Dementia

Correct answer : D. Dementia

Rivastigmine and Donepezil are cholinergic stimulators. They are used in the treatment of Alzheimer’s disease – the most common cause of dementia.

Bright light treatment has been found to be most effective in treatment of:
A. Anorexia nervosa
B. Seasonal affective disorder
C. Schizophrenia
D. Obsessive compulsive disorder

Correct answer : B. Seasonal affective disorder

Seasonal affective disorder refers to episodes of depression that occur every year during autumn or winter. Symptoms improve in spring and summer.

Mutism and akinesis in a person, who appears awake and even alert, is best described as:
A. Twilight state
B. Oneiroid state
C. Stupor
D. Delirium

Correct answer : C. Stupor

The ideal timing of radiotherapy for Wilms tumour after surgery is:
A. Within 10 days
B. Within 2 weeks
C. Within 3 weeks
D. Any time after surgery

Correct answer : A. Within 10 days

Radiotherapy is to be started within 10 days. Delay in starting radiotherapy for Wilm’s tumour can result in an increase in relapse rate.

Which one of the following imaging techniques gives maximum radiation exposure to the patient?
A. Chest X-ray
B. MRI
C. CT scan
D. Bone-scan

Correct answer : C. CT scan

Typical effective dose (mSv) for various investigations:

Chest X-ray: 0.02
CT Head: 2.3
CT Chest: 8
CT Abdomen / Pelvis: 10
Bone scan (Tc-99m): 4

Ref: Granger & Allison’s Diagnostic Radiology, 4th Edition, page 238

Which of the following inhalational agents is the induction agent of choice in children:
A. Methoxyflurane
B. Sevoflurane
C. Desflurane
D. Isoflurane

Correct answer : B. Sevoflurane

Sevoflurane has a fast, smooth and pleasant induction. Hence it is preferred in children. Sevoflurane reacts with soda lime to produce toxic products. Hence it cannot be used in a closed circuit.

Which one of the following local anesthetics belongs to the ester group?
A. Procaine
B. Bupivacaine
C. Lignocaine
D. Mepivacaine

Correct answer : A. Procaine

Other ester linked local anaesthetics:

Cocaine
Tetracaine
Benzocaine
Chlorprocaine

Which one of the following antibacterials should not be used with d-tubocurarine?
A. Norfloxacin
B. Streptomycin
C. Doxycycline
D. Cefotaxime

Correct answer : B. Streptomycin

Streptomycin can potentiate the action of d-tubocurarine and cause prolonged apnoea.

d-tubocurarine is a muscle relaxant. All aminoglycosides including streptomycin reduce ACh release from motor nerve endings. This is usually not manifested clinically. But the effect is more pronounced when given along with curare like muscle relaxants

In which of the following conditions Parakeratosis most frequently occurs?
A. Actinic keratoses
B. Seborrheic keratoses
C. Molluscum contagiosum
D. Basal cell carcinoma

Correct answer : A. Actinic keratoses

Parakeratosis refers to the persistance of cellular nuclei in the stratum corneum of skin.

A 6 month old infant had itchy erythematous papules and exudative lesions on the scalp, face, groins and axillae for one month. She also had vesicular lesions on the palms. The most likely diagnosis is:
A. Congenital shypilis
B. Seborrheic dermatitis
C. Scabies
D. Psoriasis

Correct answer : C. Scabies

A 45 year old farmer has itchy erythematous papular lesions on face, neck, ‘V‘ area of chest, dorsum of hands and forearms for 3 years. The lesions are more severe in summer and improve by 75% in winter. The most appropriate test to diagnose the condition would be:
A. Skin biopsy
B. Estimation of IgE levels in blood
C. Patch test
D. Intradermal prick test

Correct answer : C. Patch test

The history is suggestive of photodermatitis. Patch test is used for diagnosis of photodermatitis.

Patch test (photopatch test) : Here, 2 patches containing photosensitising material is applied on 2 different sites on the body. One of them is irradiated with UVA rays. The other is kept as control. Development of eczema is looked for upto 72 hours.

Analysis of results:

No development of eczema – patient does not have photodermatitis
Development of eczema under the patch irradiated with UVA – Photodermatitis
Development of eczema under both patches – Contact dermatitis

During inspiration the main current of airflow in a normal nasal cavity is through:
A. Middle part of the cavity in middle meatus in a parabolic curve
B. Lower part of the cavity in the inferior meatus in a parabolic curve
C. Superior part of the cavity in the superior meatus
D. Through olfactory area

Correct answer : A. Middle part of the cavity in middle meatus in a parabolic curve

Sensorineural deafness may be feature of all, except:
A. Nail patella syndrome
B. Distal renal tubular acidosis
C. Bartter syndrome
D. Alport syndrome

Correct answer : A. Nail patella syndrome

Which of the following conditions causes the maximum hearing loss?
A. Ossicular disruption with intact tympanic membrane
B. Disruption of malleus and incus as well tympanic membrane
C. Partial fixation of the stapes footplate
D. Ottitis media with effusion

Correct answer : A. Ossicular disruption with intact tympanic membrane (Average hearing loss is 54 dB)

Which of the following is the drug of choice for treatment of corneal ulcers caused by filamentous fungi?
A. ltraconazole
B. Natamycin
C. Nystatin
D. Fluconazole

Correct answer : C. Nystatin

SAFE strategy is recommended for the control of :
A. Trachoma
B. Glaucoma
C. Diabetic retinopathy
D. Cataract

Correct answer : A. Trachoma

The magnification obtained with a direct ophthalmoscope is:
A. 5 times
B. 10 times
C. 15 times
D. 20 times

Correct answer : C. 15 times

Brown tumors are seen in:
A. Hyperparathyroidism
B. Pigmented villonodular synovitis
C. Osteomalacia
D. Neurofibromatosis

Correct answer : A. Hyperparathyroidism

All the statements are true about exotosis, except
A. It occurs at the growing end of bone
B. Growth continues after skeletal maturity
C. It is covered by cartilaginous cap
D. Malignant transformation may occur

Correct answer : B. Growth continues after skeletal maturity

Growth of an exostosis stops with stoppage of growth at epiphyseal plate. (It is not a true neoplasm)

“Rugger Jersey Spine” is seen in:
A. Fluorosis
B. Achondroplasia
C. Renal osteodystrophy
D. Marfan’s syndrome

Correct answer : C. Renal osteodystrophy

All of the following may occur in Down’s syndrome except:
A. Hypothyroidism
B. Undescended testis
C. Ventricular septal defect
D. Brushfield`s spots

Correct answer : B. Undescended testis

The following are characteristic of autism except:
A. Onset after 6 years of age
B. Repetitive behavior
C. Delayed language development
D. Severe deficit in social interaction

Correct answer : A. Onset after 6 years of age

Features of autism usually appear before 3 years of age. (OP Ghai)

Which of the following is the principal mode of heat exchange in an infant incubator?
A. Radiation
B. Evaporation
C. Convection
D. Conduction

Correct answer : C. Convection

Which surgical procedure has the highest incidence of ureteric injury?
A. Vaginal hysterectomy
B. Abdominal hysterectomy
C. Weitheim’s hysterectomy
D. Anterior colporrhaphy

Correct answer : C. Weitheim’s hysterectomy

Which of the following conditions is least likely to present as an eccentric osteolytic lesion:
A. Aneurysmal bone cyst
B. Giant cell tumor
C. Fibrous cortical defect
D. Simple bone cyst

Correct answer : D. Simple bone cyst

Simple bone cyst presents as a central lytic lesion.

A 40 year old woman presented to the gynecologist with complaints of profuse vaginal discharge. There was no discharge from the cervix on the speculum examination. The diagnosis of bacterial vaginosis was made based upon all of the following findings on microscopy except:
A. Abundance of gram variable coccobacilli
B. Absence of Lactobacilli
C. Abundance of polymorphs
D. Presence of clue cells

Correct answer : C. Abundance of polymorphs

Bacterial vaginosis is characterised by decrease in polymorphs.

The investigation of choice in a 55 year old postmenopausal woman who has presented with postmenopausal bleeding is:
A. Pap smear
B. Fractional curettage
C. Transvaginal ultrasound
D. Ca 125 estimation

Correct answer : B. Fractional curettage

A woman having post menopausal bleeding is to be considered as a case of endometrial carcinoma unless otherwise proved. Among the options given, the best method for diagnosis of endometrial carcinoma is fractional curettage.

The tumor, which may occur in the residual breast or overlying skin following wide local excision and radiotherapy for mammary carcinoma, is:
A. Leiomyosarcoma
B. Squamous cell carcinoma
C. Basal cell carcinoma
D. Angiosarcoma

Correct answer : D. Angiosarcoma

Angiosarcoma can occur as a late complication following breast conservation surgery.

In which of the following types of breast carcinoma, would you consider biopsy of opposite breast?
A. Adenocarcinoma-poorly differentiated
B. Medullary carcinoma
C. Lobular carcinoma
D. Comedo carcinoma

Correct answer : C. Lobular carcinoma

Lobular carcinoma has higher incidence of bilaterality compared to other carcinomas

The most important prognostic factor in breast carcinoma is:
A. Histological grade of the tumor
B. Stage of the tumor at the time of diagnosis
C. Status of estrogen and progesterone receptors
D. Over expression of p53 tumour suppressor gene

Correct answer : B. Stage of the tumor at the time of diagnosis

All of the following are major complications of massive transfusion, except:
A. Hypokalemia
B. Hypothermia
C. Hypomagnesemia
D. Hypocalcemia

Correct answer :A. Hypokalemia

Massive transfusions can cause hyperkalemia due to loss of potassium from the transfused red cells.

A patient with leukemia on chemotherapy develops acute right lower abdominal pain associated with anemia, thrombocytopenia and leukopenia. Which of following is the clinical diagnosis?
A. Appendictis
B. Leukemic colitis
C. Perforation peritonitis
D. Neutropenic colitis

Correct answer : D. Neutropenic colitis

Acute right lower abdominal pain in an immunocompromised patient after chemotherapy is characteristic of neutropenic colitis.

All of the following statements regarding primary effusion lymphoma are true except:
A. It generally presents in elderly patients
B. There is often an association with HHV 8
C. The proliferating cells are NK cells
D. Patients are commonly HIV positive

Correct answer :C. The proliferating cells are NK cells

Primary effusion lymphomas are aggressive malignancies that present as serous effusion without any tumour mass.

In which of the following year the Transplantation of Human Organs Act was passed by Government of India?
A. 1994
B. 1996
C. 2000
D. 2002

Correct answer : A. 1994

Indian (economic) real GDP growth for the year 2003 is:
A. 6.0
B. 6.5
C. 7.8
D. 10.5

Correct answer : C. 7.8

According to the World Health Report 2000, India’s health expenditure is:
A. 4.8% of G.D.P.
B. 5.2% of G.D.P.
C. 6.8% of G.D.P.
D. 7% of G.D.P.

Correct answer : B. 5.2% of G.D.P.

Ethosuximide is used in the treatment of
A. Tonic-clonic seizure
B. Absence seizure
C. Myoclonic seizure
D. Simple partial seizure

Correct answer :B. Absence seizure

Ethosuximide is only used for the treatment of absence seizures.

The following statements regarding benzodiazepines are true except:
A. Binds to both GABA A and GABA B receptors
B. They have active metabolites
C. Decreases nocturnal gastric secretion in human beings
D. Extensively metabolized by CYP enzymes

Correct answer :A. Binds to both GABA A and GABA B receptors

Benzodiazepines bind only to GABA A receptors

The following are the benzodiazepines of choice in elderly and those with liver disease, except:
A. Lorazepam
B. Oxazepam
C. Temazepam
D. Diazepam

Correct answer : D. Diazepam

Diazepam has slow elimination from the body, produces active metabolites and tends to accumilate in the body with prolonged use.

A vitreous aspirate from a case of metastatic endophthalmitis on culture yields Gram-positive round to oval cells, 12-14 microns in size. The aspirate on Gram staining shows the presence of pseudohyphae. Which of the following is the most likely aetiological agent?
A. Aspergillus
B. Rhizopus
C. Candida
D. Fusarium

Correct answer :C. Candida

Candida appears as gram positive round to oval cells and pseudohyphae.

Which of the following parasitic infestation can lead to malabsorption syndrome?
A. Amoebiasis
B. Ascariasis
C. Hookworm infestation
D. Giardiasis

Correct answer : D. Giardiasis

The following diseases are associated with Epstein-Barr virus infection, except:
A. Infectious mononucleosis
B. Epidermodysplasia verruciformis
C. Nasopharyngeal carcinoma
D. Oral hairy leukoplakia

Correct answer :B. Epidermodysplasia verruciformis

Epidermodysplasia verruciformis is a rare autosomal recessive disease characterised by increased risk for development of skin cancer. There is abnormally high susceptibility to HPV infection of skin.

Mantle cell lymphomas are positive for all of the following, except
A. CD 23
B. CD 20
C. CD 5
D. CD 43

Correct answer : A. CD 23

All of the following immunohistochemical markers are positive in the neoplatic cells of granulocytic sarcoma, except:
A. CD 45 RO
B. CD 43
C. Myeloperoxidase
D. Lysozyme

Correct answer : A. CD 45 RO

CD 45 RO is a marker of T cell lymphoma and is usually absent in granulocytic sarcoma.

The type of mammary ductal carcinoma in situ (DCIS) most likely to result in a palpable abnormality in the breast is:
A. Apocrine DCIS
B. Neuroendocrine DCIS
C. Well differentiated DCIS
D. Comedo DCIS

Correct answer : D. Comedo DCIS

DCIS usually presents as mammographic calcifications. It can also present as a vague palpable mass especially in comedo DCIS.

All of the following amino acids are converted to succinyl-CoA, except:
A. Methionine
B. lsoleucine
C. Valine
D. Histidine

Correct answer : D. Histidine

Amino acids which can be converted to succinyl CoA are:

Methionine
Valine
Isoleucine
Threonine

Histidine is converted into alpha ketoglutarate instead of succinyl CoA.

The structural proteins are involved in maintaining the shape of a cell or in the formation of matrices in the body. The shape of these proteins is:
A. Globular
B. Fibrous
C. Stretch of beads
D. Planar

Correct answer : B. Fibrous

Important structural proteins are collagen, keratin, myosin etc. They are fibrous proteins.

There are more than 300 variants of human hemoglobin gene. Among these only a few are fatal. Hence, the most important factor to be conserved in a protein for its function is the:
A. Amino acid sequence
B. Ligand binding residues
C. Structure
D. Environment

Correct answer :B. Ligand binding residues

Ligand binding is most important factor with regard to conservation of protein function.

The cell bodies of orexinergic neurons are present in:
A, Locus ceruleus
B. Dorsal raphe
C. Lateral hypothalamic area
D. Hippocampus

Correct answer :C. Lateral hypothalamic area

The hyperkinetic features of Huntington’s disease are due to the loss of:
A. Nigrostriatal dopaminergic system
B. Intrastriatal cholinergic system
C. GABA-ergic and cholinergic system
D. Intrastriatal GABA-ergic and cholinergic system

Correct answer : D. Intrastriatal GABA-ergic and cholinergic system

The first reflex response to appear as spinal shock wears off in humans is:
A. Tympanic reflex
B. Withdrawal reflex
C. Neck righting reflex
D. Labyrinthine reflex

Correct answer : B. Withdrawal reflex

Intense nihilism, somatization and agitation in old age are the hallmark symptoms of:
A. Involutional melancholia
B. Atypical depression
C. Somatized depression
D. Depressive stupor

Correct answer :A. Involutional melancholia

All of the following are associated with better prognosis in schizophrenia, except:
A. Late onset
B. Married
C. Negative symptoms
D. Acute onset

Correct answer :C. Negative symptoms

Loosening of association is an example of:
A. Formal thought disorder
B. Schneiderian first rank symptoms
C. Perseveration
D. Concrete thinking

Correct answer :A. Formal thought disorder

Phosphorus 32 emits:
A. Beta particles
B. Alpha Particles
C. Neutrons
D. X rays

Correct answer : A. Beta particles

All of the following radioisotopes are used as systemic radionuclide except:
A. Phosphorus
B. Strontium
C. Iridium
D. Samarium

Correct answer : C. Iridium

Which one of the following has the maximum ionization potential?
A. Electron
B. Proton
C. Helium ion
D. Gamma ray

Correct answer :C. Helium ion

Alpha ray (helium ion) has the most ionization potential. Gamma ray has the most penetration power.

Which of the following is the muscle relaxant of choice in renal failure?
A. Rapacurium
B. Pancuronium
C. Atracurium
D. Rocuronium

Correct answer : C. Atracurium

Atracurium undergoes Hoffman degradation. So elimination does not depend upon liver or kidney function.

Which one of the following muscle relaxant has the maximum duration of action?
A. Atracurium
B. Vecuronium
C. Rocuronium
D. Doxacurium

Correct answer : D. Doxacurium

Which of the following is not an indication for endotracheal intubation?
A. Maintenance of a patent airway
B. To provide positive pressure ventilation
C. Pulmonary toilet
D. Pneumothorax

Correct answer : D. Pneumothorax

Pneumothorax should be treated with intercostal drainage / needle thoracocentesis.

A 24-year old unmarried woman has multiple nodular, cystic, pustular and comedonal lesions on face, upper back and shoulders for 2 years. The drug of choice for her treatment would be:
A. Acitretin
B. Isotretinoin
C. Doxycycline
D. Azithromycin

Correct answer : B. Isotretinoin

The clinical picture is diagnostic of acne vulgaris. Isotretinoin is used in severe intractable acne.

Pterygium of nail is characteristically seen in:
A. Lichen planus
B. Psoriasis
C. Tinea unguium
D. Alopecia areata

Correct answer : A. Lichen planus

A patient had seven irregular hyperpigmented macules on the trunk and multiple small hyperpigmented macules in the axillae and groins since early childhood. There were no other skin lesions. Which is the most likely investigation to support the diagnosis?
A. Slit lamp examination of eye
B. Measurement of intraocular tension
C. Examination of fundus
D. Retinal artery angiography

Correct answer : A. Slit lamp examination of eye

The irregular hyperpigmented macules since early childhood are probably cafe au lait macules. Presence of 6 or more cafe au lait macules is diagnostic of neurofibromatosis type I even when cutaneous neurofibromas are absent. Lisch nodules on the iris can be demonstrated by slit lamp examination and it helps support the diagnosis.

Which is the investigation of choice in assessing hearing loss in neonates?
A. Impedance audiometry
B. Brainstem Evoked Response Audiometry (BERA)
C. Free field audiometry
D. Behavioral audioimetry

Correct answer :B. Brainstem Evoked Response Audiometry (BERA)

A 30 year old male is having Attic cholesteatoma of left ear with lateral sinus thrombophlebitis. Which of the following will be the operation of choice?
A. lntact canal wall mastoidectomy
B. Simple mastoidectomy with Tympanoplasty
C. Canal wall down mastoidectomy
D. Mastoidectomy with cavity obliteration

Correct answer : C. Canal wall down mastoidectomy

Attic cholesteatoma is an indication for modified radical mastoidectomy. (Canal wall down mastoidectomy)

All the following polyps are premalignant except
A. Juvenile polyposis syndrome
B. Famillial polyposis syndrome
C. Juvenile polyp
D. Peutz jegher syndrome

Correct answer : C. Juvenile polyp

Which of the following statements related to gastric injury is not true?
A. Mostly related to penetrating trauma
B. Treatment is simple debridement and suturing
C. Blood in stomach is always related to gastric injury
D. Heals well and fast

Correct answer : C. Blood in stomach is always related to gastric injury

Preferred incision for abdominal exploration in Blunt injury abdomen is:
A. Always midline incision
B. Depending upon organ
C. Transverse incision
D. Paramedian

Correct answer : A. Always midline incision

All of the following statements about Gynaecomastia are true except:
A. Subcutaneous mastectomy is the initial treatment of choice
B. Seen in liver disease
C. There may be estrogen – testosterone imbalance
D. Can be drug induced

Correct answer : A. Subcutaneous mastectomy is the initial treatment of choice

The following is a marker of Paget’s disease of breast
A. S- 100
B. HMB 45
C. CEA
D. Neuron specific enolase

Correct answer : C. CEA (carcinoembryonic antigen)

All of the following are true about Lymphoma of the thymus except:
A. More common in females
B. Slow growing
C. Clinically confused with undifferentiated tumours
D. May present with respiratory distress and dysphagia

Correct answer : B. Slow growing

Which of the following is not a histological variant of thyroid neoplasm?
A. Follicular
B. Merkel cell
C. lnsular
D. Anaplastic

Correct answer : B. Merkel cell

Treatment of severe ulnar neuritis in borderline tuberculoid leprosy is:
A. MDT only
B. MDT + steroid
C. Wait and watch
D. MDT + thalidomide
(MDT = Multi Drug Therapy)

Correct answer : B. MDT + steroid

All of the following organs may be involved in Leprosy except:
A. Uterus
B. Ovary
C. Testes
D. Eye

Correct answer : A. Uterus

Female genital tract is seldom involved in leprosy. Ovary is the most common site of leprosy in the female genital tract.

Which of the following is the classical CSF finding seen in Tuberculous meningitis?
A. Increased protein, decreased sugar, increased lymphocytes
B. Increased protein, sugar and lymphocytes
C. Decreased protein, increased sugar and lymphocytes
D. Increased sugar, protein and neutrophils

Correct answer : A. Increased protein, decreased sugar, increased lymphocytes

The following are components of Brown Sequard syndrome except :
A. Ipsilateral extensor plantar response
B. lpsilateral pyramidal tract involvement
C. Contralateral spinothalamic tract involvement
D. Contralateral posterior column involvement

Correct answer : D. Contralateral posterior column involvement

In brown sequard syndrome ( hemisection of spinal cord ), there is ipsilateral posterior colum involvement.

All of the following statements are true about Benedict’s syndrome, except:
A. Contralateral tremor
B. 3rd nerve palsy
C. Involvement of the penetrating branch of the basilar artery
D. Lesion at the level of the pons

Correct answer : D. Lesion at the level of the pons

Benedict’s syndrome is a dorsal mid brain syndrome.

Millard Gubler syndrome includes the following except :
A. 5th nerve palsy
B. 6th nerve palsy
C. 7th nerve palsy
D. Contralateral hemiparesis

Correct answer : A. 5th nerve palsy

Millard Gubler syndrome (ventral pontine syndrome) is characterised by:

Ipsilateral paresis of lateral rectus (6th nerve)
Ipsilateral UMN type of facial weakness (7th nerve)
Contralateral hemiplegia (pyramidal tract)

Pontine Stroke is associated with all except :
A. Bilateral pin point pupil
B. Pyrexia
C. Vagal palsy
D. Quadriparesis

Correct answer : C. Vagal palsy

Vagal palsy is not a component of pontine stroke.

Increased ICT is associated with all except
A. Paraparesis
B. Abducent paralysis
C. Headache
D. Visual blurring

Correct answer : A. Paraparesis

Increased ICT alone does not cause paraparesis. It can occur if the increased ICT is due to a mass lesion in the parasagittal region.

The following are components of Kawasaki disease except,
A. Purulent conjunctivitis
B. Pedal edema
C. Truncal rash
D. Pharyngeal congestion

Correct answer : A. Purulent conjunctivitis

Non purulent conjunctivitis is seen in Kawasaki disease.

All of the following true about Raynaud’s disease except:
A. More common in females
B. Positive Antinuclear Antibodies
C. Most common cause of raynaud’s phenomenon
D. Has good prognosis

Correct answer : B. Positive Antinuclear Antibodies

Antinuclear Antibodies are seen only in secondary causes of Raynaud’s phenomenon. Raynaud’s disease refers to idiopathic Raynaud’s phenomenon.

Best marker for drug induced lupus is.
A. Anti histone antibodies
B. Anti ds DNA
C. Anti neutrophil antibody
D. Anti Smith antibody

Correct answer : A. Anti histone antibodies

Anti histone antibodies are the best markers for drug induced lupus.

Deposition of Anti ds DNA antibody in kidney, skin, choroid plexus and joints is seen in:
A. SLE
B. Goodpasture syndrome
C. Scleroderma
D. Raynauds disease

Correct answer : A. SLE

Anti ds DNA antibody is specific for SLE.

C- ANCA positivity indicates antibody formed against
A. Proteinase 3
B. Myeloperoxidase
C. Cytoplasmic antinuclear Antibody
D. Anti centromeric Antibody

Correct answer : A. Proteinase 3

Proteinase 3 is the major C ANCA antigen.

Hypophosphatemia is seen in all except:
A. Acute renal failure
B. Resolving phases of diabetic ketocidosis
C. Respiratory alkalosis / COPD
D. Chronic alcoholism

Correct answer : A. Acute renal failure

Hyperphosphatemia is seen in renal failure.

SIADH is associated with the following drug:
A. Vincristine
B. Erythromycin
C. 5 — FU
D. Methotrexate

Correct answer : A. Vincristine

Treatment of choice for Zollinger Ellison Syndrome (ZES) is:
A. PPI
B. Somatostatin analogues
C. Streptozocin
D. Sucralfate

Correct answer : PPI

Which of the following endocrine tumors is most commonly seen in MEN I?
A. lnsulinoma
B. Gastrinoma
C. Glucagonoma
D. Somatostatinoma

Correct answer : Gastrinoma

A 60 year old woman presents with generalized edema, skin ulceration and hypertension. Urine examination shows subnephrotic proteinuria (<2gm) and microscopic hematuria. Serum complement levels are decreased and she is positive for anti-hepatitis C antibodies. The likely diagnosis is :

A. PSGN
B. Essential mixed cryoglobulinemia
C. Membrano proliferative glomerulonephritis
D. Focal segmental glomerulosclerosis

Correct answer : Essential mixed cryoglobulinemia

The test used to diagnose Dubin Johnson syndrome is:
A. Serum transminases
B. Bromsulphalein test (BSP)
C. Hippurate test
D. Gamma glutamyl transferase level

Answer: B. Bromsulphalein test (BSP)

A 30 year old man presents with generalized edema and hypertension. Urine examination shows subnephrotic proteinuria (< 2gm) and microscopic hematuria. Serum complement levels are decreased and he is positive for antihepatitis C antibodies. The most likely diagnosis is:
A. Post streptococcal Glomerulonephritis (PSGN)
B. Mixed cryoglobulinemia
C. Membranoproliferative glomerulonephritis (MPGN)
D. Focal symental Glomerular sclerosis (FSGS)

Answer: C. Membranoproliferative glomerulonephritis (MPGN) > B. Mixed cryoglobulinemia

Most common cause of nephrotic range proteinuria in an adult is:
A. Diabetes Mellitus
B. Amyloidosis
C. Hypertensive nephropathy
D. Wegner`s Granulomatosis

Answer: A. Diabetes Mellitus

A patient with H. Pylori infection is treated with drugs. The best method to detect presence of residual H.Pylori infection in this person is :
A. Rapid urease test
B. Urea breath test
C. Endoscopy and biopsy
D. Serum anti H.Pylori titre

Ans: B. Urea breath test

Recommended test for confirmation of H.pylori eradication – urea breath test.

All of the following drugs are commonly used in regimens against H. pylori except:
A. Oxytetracycline
B. Amoxicillin
C. Bismuth Subcitrate
D. Omeprazole

Answer : A. Oxytetracycline

Oxytetracycline is not a recommended drug for treatment of H.pylori.

A patient presents with decreased vital capacity and total lung volume. What is the most probable diagnosis?
A. Bronchiectasis
B. Sarcoidosis
C. Cystic fibrosis
D. Asthma

Answer : B. Sarcoidosis

A restrictive lung disease causes decrease in vital capacity and total lung volume. Among the options provided, sarcoidosis is the only one causing a restrictive lung disease.

A 70 year old man develops pneumonia and septicemia. Patient goes into renal failure and has a BP of 70/50 mm of Hg. Drug that should be used to maintain BP is:
A. Adrenaline
B. Ephedrine
C. Phenylephrine
D. Nor epinephrine

Answer : D. Nor epinephrine

The patient is in septic shock. Dopamine is the recommended drug for maintenance of BP in a case of septic shock. If there is no response, nor epinephrine should be tried. Since nor epinephrine is the only one given among the options, it is the best answer.

A patient presents with acute anterior wall infarction and hypotension. Which will be the immediate treatment modality for this patient?
A. Intra aortic balloon counter pulsation
B. Anticoagulation
C. Thrombolytic therapy
D. Angiography and Primary angioplasty

Answer: D. Angiography and Primary angioplasty

Angiography and primary angioplasty is the best modality of treatment in case of ST eleveation MI presenting within 12 hours.

You have diagnosed a Patient clinically as having SLE and ordered 6 tests out of which 4 tests have come positive and 2 are negative. To determine the probability of SLE at this point, you need to know
A. Prior probability of SLE, sensitivity and specificity of each test
B. Incidence of SLE and the predictive value of each test
C. Incidence and prevalence of SLE
D. Relative risk of SLE in the patient

Answer: A. Prior probability of SLE, sensitivity and specificity of each test

A study was conducted to test if use of herbal tea played any role in prevention of common cold. Data was collected on the number of people who developed cold and those who did not develop cold and this data was tabulated in relation to whether they consumed Herbal tea or not. The analytical test of choice to test this study is:
A. Student- t test (unpaired)
B. Student- t test (paired)
C. Chi square test
D. ‘Z’ test

Correct answer : C. Chi square test

The test required is a non parametric test (to compare qualitative data). Chi square test a frequently used non parametric test.

A data is arranged as satisfied, very satisfied, dissatisfied. This is a
A. Nominal data
B. Ordinal data
C. Interval data
D. Ratio data

Correct answer : B. Ordinal data

All of the following are true about cluster sampling except
A. Samples are similar to those in Simple Random sampling
B. ls a Rapid and simple method
C. The sample size may vary according to study design
D. It is a type of probability sample

Correct answer : A. Samples are similar to those in Simple Random sampling

Sampling is cluster sampling is different from that in simple random sampling.

Which of the following is not an aryl phosphate?
A. TIK 20
B. Malathion
C. Parathion
D. Follidol

Correct answer : B. Malathion

Malathion is an alkyl phosphate.

The drug of choice for mushroom poisoning is:
A. Atropine
B. Physostigmine
C. Adrenaline
D. Carbachol

Correct answer : A. Atropine

Atropine is useful in early onset mushroom poisoning. It counteracts the muscarinic symptoms.

Methotrexate is used in high doses in:
A. Osteosarcoma
B. Retinoblastoma
C. Rhabdomyosarcoma
D. Ewing’s sarcoma

Correct answer : A. Osteosarcoma

Methotrexate is used in high doses (with leucovorin rescue) for treatment of osteosarcoma, hematological malignancies and brain tumours.

The following drug acts by hypomethylation
A. Gemcitabine
B. Capecitabine
C. Decitabine
D. Cytosine arabinoside

Correct answer : C. Decitabine

Decitabine acts by causing hypomethylation. It inhibits DNA methyl transferase.

All are true about immunosuppressants except?
A. Tacrolimus inhibits calcineurin pathway
B. Steroids binds to cytosolic receptors and heat shock proteins
C. Mycophenolate inhibit purine synthesis via GMP dehydrogenase
D. Sirolimus will block kinase in the IL 2 receptor pathway

Correct answer : C. Mycophenolate inhibit purine synthesis via GMP dehydrogenase

Mycophenolate inhibit purine synthesis via IMP (Inosine monophosphate) dehydrogenase

Nevirapine belongs to the following group
A. Non Nucleoside Reverse Transcriptase Inhibitor (NNRTI)
B. Protease inhibitor
C. Fusion inhibitor
D. Nucleoside reverse transcriptase inhibitor (NRTI)

Correct answer : A. Non Nucleoside Reverse Transcriptase Inhibitor (NNRTI)

Nevirapine and Efavirenz are Non Nucleoside Reverse Transcriptase Inhibitors (NNRTI)

Rabies is identified by
A. Guarneri bodies
B. Negri bodies
C. Cowdry A bodies
D. Cowdry B bodies

Correct answer : B. Negri bodies

Guarneri bodies – Vaccinia
Negri bodies – Rabies
Cowdry A bodies –  Herpes / yellow fever
Cowdry B bodies – Adenovirus / pollovirus

Which of the following statement is not true regarding Chlamydia Trachomatis:
A. Elementary body is metabolically active
B. It is biphasic
C. Reticulate body divides by binary fission
D. Inside the cell it evades phagolysome

Correct answer : A. Elementary body is metabolically active

Reticulate body is metabolically active.

With reference to bacteriodes fragilis, the following statements are true except:
A. Bacteriodes fragilis is a frequent anaerobe isolated from clinical samples
B. Bacteriodes fragilis is not uniformly sensitive to metronidazole
C. The LPS formed by Bacteriodes fragilis is structurally and functionally different from the conventional endotoxins
D. Shock and DIC are common in bacteroides bacteremia

Correct answer : D. Shock and DIC are common in bacteroides bacteremia

Shock and DIC are not common in bacteroides bacteremia.

Botulinum affects all except:
A. Neuromuscular junction
B. Preganglionic junction
C. Post ganglionic nerves
D. CNS

Correct answer : D. CNS

CNS is not involved in botulism.

Brucella is transmitted by all expect:
A. Through the placenta of animals
B. Aerosol transmission
C. Person to person
D. Eating uncooked food

Correct answer : C. Person to person

There is no person to person transmission in brucella.

A person working in an abattoir presented with a papule on hand, which turned into an ulcer. Which will best help in diagnosis?
A. Trichrome Methylene Blue
B. Carbol Fuschin
C. Acid Fast Stain
D. Calcoflour White

Correct answer : A. Trichrome Methylene Blue

The clinical features are suggestive of cutaneous anthrax. Trichrome methylene blue is used to demonstrate McFadyean’s reaction which is characteristic of Bacillus anthracis.

An elderly male patient presented with fever, chest pain and dry cough. Sputum culture showed growth on Charcol Yeast Extract Medium. The organism is
A. H. influenza
B. Moraxella cartarrhalis
C. Legionella
D. Burkholderia cepacia

Correct answer : C. Legionella

Legionella grows only in special media like Charcol Yeast Extract Medium

All the following statements are true about V. cholera 0139 except:
A. Clinical manifestations are similar to Eltor strain
B. First discovered in Chennai
C. Epidemiologically indistinguishable from O1 Eltor strain
D. Produces O1 Lipo polysaccharide

Correct answer : D. Produces O1 Lipo polysaccharide

V. cholera 0139 does not synthesise O1 Lipopolysaccharide

Regarding synthesis of triacyl glycerol in adipose tissue, all of the following are true except:
A. Synthesis from Dihydroxyacetone phosphate
B. Enzyme Glycerol kinase plays an important role
C. Enzyme Glycerol 3 phosphate dehydrogenase plays an important role
D. Phosphatidate is hydrolyzed

Correct answer : B. Enzyme Glycerol kinase plays an important role

Glycerol kinase is not involved in the triglyceride synthesis in adipose tissue.

Dinitrophenol causes:
A. Inhibition of ATP synthase
B. Inhibition of electron transport
C. Uncoupling of oxidation and phosphorylation
D. Accumulation of ATP

Correct answer : C. Uncoupling of oxidation and phosphorylation

Dinitrophenol uncouples oxidation and phosphorylation in the respiratory chain. The hydrogen ions re enter the mitochondrial matrix without ATP synthesis. The energy is released as heat.

All of the following occur in mitochondria except:
A. Citric acid cycle (Kreb’s cycle)
B. Glycogenolysis
C. Fatty acid oxidation
D. Electron transport chain

Correct answer : B. Glycogenolysis

Glycogenolysis occurs in the cytoplasm.

Other metabolic reactions in the cytosol – glycolysis, glycogenesis, cholesterol synthesis, HMP shunt, bile acid synthesis

Which of the following enzymes is stable at acidic pH
A. Pepsin
B. Trypsin
C. Chymotrypsin
D. Carboxypeptidase

Correct answer : Pepsin

Pepsin is formed when pepsinogen is activated by hydrochloric acid in the stomach. The optimum pH for pepsin action is 1.6 to 3.2.

Mercury will affect which part of the renal tubule
A. PCT
B. DCT
C. CT
D. Loop of Henle

Correct answer : PCT

Mercury poisoning (also known as hydrargyria or mercurialism) is a disease caused by exposure to mercury or its compounds.

Burtonian line is seen with poisoning of
A. Mercury
B. Lead
C. Arsenic
D. Zinc

Correct answer : Lead

Burtonian line, also known as lead line is a clinical manifestation of chronic lead poisoning